You are on page 1of 148

ENGLISH

USAGE
(Vol.-I)

This book is a part of set, not be sold separately.


INTRODUCTION
The book in your hand contains several chapters on English usages keeping in
view the types of questions asked in CAT exams. The chapters are designed with a
well chalked out method. Every chapter contains the theoretical part followed by
some examples with their solutions. At the end of the chapters, you have a few
exercises with their explanatory answers to check your understanding of the chapter.
For further practice, you will be given one book specifically on exercises.
To get the best advantage of the book, you should first read the theoretical part
carefully. In this part you will come to know the methods and techniques to solve
certain type of questions. As time management is the prime requirement of CAT
exam, you cannot afford to answer the question leisurely. It is essential, thus, to be
aware of the short routes to solve the answer. The theoretical part of the book can
be quite handy in this respect. The techniques are further explained with some
example and their elaborate solutions. You are required to first read the examples
and try to answer them yourself. You can then see in ‘solution’, the methods by
which the example is solved. After finishing the chapter you can check your grasp
on it by solving the exercises. For your convenience, exercises are given explanatory
answers, but please do not look at the answers before you have solved the questions
yourself.
Except Reading Comprehension for which you are provided a separate book,
the book covers all types of questions from English requisite for CAT preparation.
It has Critical Reasoning, Fill in the Blanks, Word Related Questions and so on.
You will have ample familiarization with each type of question in detail. Besides this,
it also has some lists of words and phrases that can prove useful in your preparation.
For example, there is a list of Confusing Words, words which an average user use
interchangeably and makes mistake. There is also a list of Near Synonyms. Some
words appear synonymous to one another, but have light shade of difference in
their meaning depending on the context. So, an intelligent knower of English won’t
use one word for another.
Finally, we hope that the book proves sufficiently helpful in your preparation of
CAT exams. We are desirous of further suggestions from the students.

BEST OF LUCK
Contents
1. Critical Reasoning ... 7 - 28
2. Jumbled Paragraphs ... 29 - 45
3. Fill in the Blanks ... 46 - 72
4. Syllogism ... 73 - 102
5. Question Related to Words ... 103 - 117
6. Common Misspellings ... 118 - 127
7. Synonyms ... 128 - 137
Answers ... 138 - 140
Explanations ... 141 - 147
1
Critical Reasoning

The primary object of the critical reasoning questions is to test the skills in
constructing and evaluating arguments. Such questions are normally followed by
certain statements or small passage that, most of the times, has a main idea which
the writer tries to support and strengthen with certain arguments.
The ability to think critically requires a number of skills. Firstly, there is familiarity
with the structure and nature of arguments and reasoning. Secondly, one should to
be able to put arguments into a standard analyzable format, as well as discern how
the several parts relate to each other. The next skill is the ability to locate and assess
the strengths and weaknesses of the argument. Finally, the entire process must then
be viewed in light of the context, goals and values of the arguer.

BASIC STRUCTURE OF A STATEMENT/PASSAGE


PREMISE
It is the evidence given to support a conclusion. It can be identified with words
such as “if”, “given that”, “since”, “because”, “in view of”, “suppose” etc., which
signal the presentation of evidence and reasons in support of a fact or claim.

ASSUMPTION
An assumption is an implicit statement that fills the gap in the argument. It
supports the conclusion; but may or may not be convincing enough to hold the
argument and conclusion valid.
For Example:
“The ground was wet, so it must have been raining.”
Here, “The ground was wet” is premise. The conclusion “it must have been
raining” is based on the assumption of the writer that ground will be wet only when
it rains; where as it could have been wet for a variety of other reasons. Thus, here
the argument and conclusion are invalid or can be very easily contradicted.
ENGLISH USAGE

Another statement:
“The ground was wet, because it was raining.”
Here, “it was raining” is the premise, from which we can infer that when
ever it rains, the ground becomes wet.

ARGUMENT
It is a statement or a supporting reason meant to further strengthen the assertion.
It is a sequence of two or more phrases, clauses, sentences or statements, one of
which is a claim or conclusion, which follows the Premises.
An argument is composed of two kinds of statements: (1) the conclusion (main
idea) is that statement which follows from the other statements and (2) the reasons
(evidence) are those statements, which are intended to support the conclusion or to be
precise, the premises.
The second example stated above makes a valid argument.
Another example of a valid argument can be “The ground was wet, so it might
have been raining.”

CONCLUSION
A conclusion sums up the main idea with which the statement/ paragraph has
been written. It is stated in the passage but does not have to necessarily be at the
end of the argument and is often preceded by words such as “thus”, “hence”,
“therefore” etc.

INFERENCE
Webster dictionary defines inference as ‘the act of passing from one proposition,
statement, or judgment, considered as true to another whose truth is believed to
follow from that of the former’. It must be clear to you now that inferences are
unstated conclusions drawn from a stated fact or proposition.
Inferences are evidence-based guesses. They are the conclusions a reader
draws about the unsaid, based on what is actually said. We draw inferences in our
everyday life. If your best friend comes in from a blind date and looks very cheerful,
you would probably infer the date was a success. You have concluded that the date
was a success though your friend has said nothing about it

8
CRITICAL REASONING

CONNECTING EVENTS TO DRAW CONCLUSIONS


Arguments frequently contain a number of premises and possibly more than
one conclusion. Therefore, it is necessary to classify and connect things and events
in order to analyze the arguments.
For example:
“Rachna overslept, which caused her to be late for work; therefore she drove
fast and met with an accident.”
Above statement might prima facie seem to conclude that since Rachna
overslept, she met with an accident. Which is absolutely wrong and we need to
connect the right events too get the right conclusion. Here Rachna met with an
accident since she was driving fast (and not because she overslept!!) and the
underlying assumption is that while driving she must have been not just fast but
careless as well.
Further one needs to identify what the writer is actually trying to prove. What
is the scope and extent of his claim? We have already seen in earlier example the
difference between the claims made in statements
“The ground was wet, so it must have been raining.”
“The ground was wet, so it might have been raining.”
Here, the claim in the second sentence is limited to only a certain extent i.e.
there is a probability (no matter how small) that it could have been raining. Thus,
writer’s argument is valid.
Descriptive words in a passage are also used to limit or expand the claims made
by the writer:
For Example:
“Prisoners in Beur Jail rioted today because they were angry about their
conditions.”
Here the word “Prisoners” indicates that there was more than one (may or may
not be “all the prisoners”) prisoner that rioted. Also, the claim is about the prisoners
in Beur Jail only and can’t be extended to all the prisoners in other jails as well. Also,
writer assumes that the reason for the riot was that the prisoners were angry only
about their conditions. However, this assumption can be incorrect as there could be
other reasons as well. There might have been fight between the two gangs of
prisoners.
9
ENGLISH USAGE

We have to be careful not to blindly assume the writer’s assumption to be


correct even if it seems to be true. There should be enough supporting evidence to
support the assumption and thus the argument and conclusion.

HOW TO TACKLE SUCH QUESTIONS


1. Read the question carefully before reading the passage.  Know what you should
be looking for before you begin reading the passage. You will want to approach
the passage, depending on whether you are asked to destroy an argument, or
to find the best conclusion to the passage. But make sure that you do not read
the answer choices as the test makers will deliberately include answer choices
that give ‘right’ answers to wrong questions.
2. Identify the passage’s assumptions and conclusion. Sometimes you need to
infer the unstated conclusions. Look for these patterns:
A paragraph may start off with its conclusion in the first sentence, and then
give several sentences to support that conclusion. This means you will not be
able to look for transition words such as “consequently”, “hence”, and “as a
result” that are commonly used to indicate the conclusion.
An assumption - if the assumption is not true, it follows that the conclusion is
not true. For identifying assumptions, read each sentence in the passage
individually, and then ask yourself if the conclusion would still be true if this
one sentence was incorrect.
3. Read every answer choice.  Don’t settle for choosing the first one that seems
right to you. You may find another answer choice that is even better than the
one you initially selected. Eliminate the ones you know are wrong. Carefully
analyze the remaining choices with a focus on identifying the one that presents
the most relevant arguments and raises the most relevant issues.
4. Utilize process of elimination.  When the test taker is asked to identify the
statement that does the best job of strengthening or weakening an argument,
there is almost always at least one answer choice that will do the opposite. If
you have read the question carefully, you will be able to quickly eliminate these
choices.
5. Become comfortable at “working backwards” on these questions.  “Working
backwards” – inserting each answer choice into the text and and seeing if the
passage still makes sense – is an excellent technique to fall back on if you get
10
CRITICAL REASONING

stuck on a critical reasoning question. However, it can be time consuming. You


may need to re-read a passage twice or even thrice, inserting a different answer
choice each time, before you find the choice that seems right to you.
6. Never choose an answer simply because it is true.  The answer choice must be
a logical extension of the argument made in the passage.
7. Ignore decoys.  Often, passages contain extraneous sentences and information.
Learn to separate these decoys from the rest of the passage so they won’t
distract you from the content that is important.
8. Avoid answer choices that make absolute statements.  Absolute statements
are those that use words such as “always” and “must.” The test writers are
very biased against these types of statements.
The following examples can help you handle such questions in a better ways.
Example 1: Advertisement: Value Search is gaining more subscribers each
year than any other business news service. Value Search offers the most up-to-
date international business news, and the most comprehensive company
information needed to make wise investment decisions. Value Search is the only
service devoted exclusively to international business news and the financial
analysis of corporations. So, by choosing a financial news service other than
Value Search, you are doing your company a disservice.
Which of the following is an assumption of the argument in the advertisement
above?
(1) A subscription to Value Search is not appreciably more expensive than a
subscription to standard business magazines or newspapers.
(2) A significant portion of any company’s business involves international
trade or investing in other companies.
(3) Value Search has more subscribers than other business news services.
(4) Choosing a financial news service other than Value Search is worse than
choosing no financial news service at all.
Solution : We’re looking for an assumption that connects evidence and
conclusion. First, read the stimulus and identify the argument. How are the evidence
and conclusion different from each other? Try to work out the assumption that
holds them together.

11
ENGLISH USAGE

The advertisement contains an argument built on three pieces of evidence:


* Value Search is growing fastest;
* Value Search offers the most up-to-date international business news, and
the most comprehensive company information needed to make wise
investment decisions;
* Value Search is the only service devoted exclusively to these two types of
news.
Based on this evidence, the advertisement concludes that subscribing to any
other financial news service would be a disservice to your company. There are many
assumptions here. But the most crucial assumption is that the two types of news
Value Search provides must be useful to any company.
Option 2 explicitly states the latter assumption. So, the answer is 2]
Example 2: Staff members at the Agra Detention Center typically oversee
students’schedules and make all final decisions regarding the required activities in
which students participate. Students are permitted, however, to make their own
decisions regarding how they spend their free time. Therefore, students should be
permitted to make their own decisions regarding the elective courses that they wish
to take.
The conclusion above would be more reasonably drawn if which of the following
were inserted into the argument as an additional premise?
(1) Decisions regarding required activities are more important than decisions
regarding the elective courses that students take.
(2) Students are more willing to take elective courses than to participate in
required Center activities.
(3) Required activities contribute more to the students’ rehabilitation than do
their free-time activities.
(4) When compared for decision-making purposes, elective courses are more
like free-time activities than required activities.
Solution : Since students at the Agra Detention Center make their own decisions
about free time, they should also make their own decisions about elective courses.
The argument assumes that, for these students, decisions about what to do with
free time are similar to decisions about what electives to take in school. Since we’re
looking for a premise to complete the argument, look for a choice that provides

12
CRITICAL REASONING

evidence that these things are similar. Option 4 provides the evidence we need. If it
is true that elective courses are more akin to free time than to required activities,
then the conclusion that students should be able to choose their own electives is
more reasonable.
Now, look at the options one by one:
(1) doesn’t serve as the premise we seek: It’s not the importance of the decision
that’s at issue here—it’s who should have the power to make the decision.
This aspect of “importance” is outside the scope of the question.
(2) is irrelevant: Students’ relative willingness to take the different types of
courses doesn’t matter, again the argument concerns only who should
make the decisions regarding activities.
(3) may explain why staff members oversee decisions about required activities,
but it tells us nothing about why students should be allowed to choose
their own elective courses in fact, it doesn’t refer to elective courses at all.
Only 4] gives us the missing premise we need. So, the answer is 4]
Example 3: Suicide bombings are no answer, says United States President
George W. Bush, who probably has not a clue about the despair of the deprived, as
well as by many others who do have a clue and are more sensitive. No right-thinking
person will refute that the killing of innocents sitting in a Tel Aviv cafe or buying
cakes in a Jerusalem bakery is wrong. But in the current catch-all atmosphere of the
war against terror, the nuances should not be lost sight of. These Palestinians are
not people sitting in caves in Afghanistan, being brainwashed with all sorts of
delusory messages and then setting off to kill people they neither know nor have
reason to dislike. These acts tell of their exasperation they are having after long and
fruitless struggle against the occupation by Israel.
Which statement is the suitable inference from the passage above?
(1) Terrorism in general should be dealt sympathetically because they are
victimizers who have been victimized themselves.
(2) Palestinians are more sensible and peace loving people than those of
Afghanistan
(3) The suicide bombers of Palestine are in a desperate condition after waging
long war against occupation.
(4) It is ethically bad to kill people sitting in cafe or buying cakes.

13
ENGLISH USAGE

Solution : The author has a sympathetic attitude towards the Palestinian suicide
bombers but the passage does not give any indication that he feels the same for the
terrorists in general. We can eliminate choice 1] then.
The author considers the case of Palestinian terrorists understandable who are
not like brainwashed Afghani terrorists but their cause is genuine and
understandable. But he does not hold that Palestinians are more peace loving than
Afghanis. We can rule out option 2]
It may be deduced that to kill innocent people is ethically bad. But this is NOT
what the author has set out to project. We should do away with option 4]
The statement in option 3] is unmistakably in accordance with the author’s
opinion. So, the answer is 3]
Example 4: With a scarcely concealed sneer, Hitchens concluded with the
observation that Chomsky had “lost or is losing the qualities that made him a great
moral and political tutor in the years of the Indo-china war and that enabled him to
write such monumental essays as his critique of the Kahan Commission on (the)
Sabra and Shatila (massacres of Palestinian refugees) or his analysis of the situation
in East Timor.” For the crowds that have been thronging Chomsky’s public
engagements in India, this observation must surely seem both trivial and irrelevant.
Unlike others who have claimed the mantle of the moral tutor in the West, Chomsky
has never departed from the fundamental principle that no event or entity can be
spared the rigors of rational analysis.
Which of the following is true of the passage?
(1) Chomsky’s deployment of rigorous analysis of the situations has made
him a great moral and political leader.
(2) The public engagement of Chomsky substantiates the opinion of Hitchens.
(3) The author seems to be agreeing with Hitchens.
(4) The author is in disagreement with Hitchens.
Solution: The passage mentions about ‘rigours of rational analysis’ of Chomsky,
but it does not state that this is what has made him famous. Thus, 1] is not the
correct answer. The public engagement does not substantiate but belies
Hitchens. 2] has to be ruled out then. Option 3] is also ruled out because the author
is not in agreement with Hitchens. So, the answer is 4]

14
CRITICAL REASONING

EXERCISE – 1
(1) Stock analyst: “We believe Company A’s stock will appreciate at 35% a
year for the next 5 to 7 years. Company A just became the leader in its industry and
we expect its sales to grow at 8% a year.” Commentator: “But how can the stock’s
price be expected to grow more quickly than the company’s underlying sales?”
Which of the following facts would best support the stock analyst?
(1) The stock price of the company also depends on factors like its management
and it is there that the company scores very well.
(2) The company just won a patent on a new product.
(3) Company A’s stock is currently overvalued by a significant amount.
(4) The 5 to 7 year time frame is too long for anyone to accurately forecast.
(2) Acme brand aspirin claims to be the best headache relief available in the
market today. To prove this claim, Acme called 10 people and asked them their
thoughts on headache relief products. All 10 of them stated that they unequivocally
use Acme brand aspirin on a regular basis and that they believe it to be the best
headache relief available on the market today.
Which of the following would most weaken this argument?
(1) Acme brand aspirin is highly addictive.
(2) The 10 people called were married to the company’s top 10 executives, and
they were coached on what to say.
(3) Most people choose to suffer silently through their headaches and take
no medicines whatsoever.
(4) An independent company conducted this survey.
(3) My neighbor’s dogs bark and howl every time their owner lets them outside.
My friend told me that dogs tend to bark and howl when they see birds resting in the
top branches of their favourite trees. I personally believe they bark and howl because
they enjoy disrupting my meditations.
Which of the following can be inferred from the preceding passage?
(1) The owners must abuse their dogs.
(2) The dogs will bark and howl at 3 a.m. if they are outside at that time
(3) There are many pedestrians who walk by this neighbor’s house, and the
dogs are starving for attention.
(4) The dogs enjoy being outside.
15
ENGLISH USAGE

(4) Most citizens are very conscientious about observing a law when they can
see the reason behind it. For instance, there has been very little need to actively
enforce the recently implemented law that increased the penalty for motorists caught
leaving a gas station without paying for gas they had pumped into their vehicles.
This is because citizens are very conscientious of the high cost of gasoline and
they know that stealing gas will only further increase the price of gasoline for
everyone.
Which of the following statements would the author of this passage is most
likely to believe?
(1) The increased penalty alone is a significant motivation for most citizens to
obey the law.
(2) There are still too many inconsiderate citizens in the local community.
(3) High gasoline prices can be brought down if everyone does his or her part
and pays for the gasoline they use at the pumps.
(4) Society should make an effort to teach citizens the reasons for its laws.
(5) Japan wants the G-4 to strengthen its activities further in the coming months
when lobbying for a seat in a restructured Security Council is expected to intensify.
Japanese Ambassador to India Yasukuni Enoki, in a briefing to the media. He also
added that all aspirants for a Security Council seat should be “realistic and flexible”
on the issue of having a “veto power”.
What inference can be drawn from Yasukuni’s statement with regard to “veto
power”?
(1) The aspirants should not accept Security Counsil seat without having
veto power.
(2) Veto power is not an issue, they can give it up provided they are offered
permanent seat.
(3) The G-4 should be pressurized to advocate veto power for the aspirants.
(4) The aspirants should demand veto power, but should not be unrelenting
about it.
(6) Some considerations seem to have wrought the unfortunate shift in India’s
stand. Firstly, an obsessive fear that weakening Gyanendra would lead to Maoist
infiltration from Nepal and a rise in naxalite violence. Secondly, the King’s plea that
he is running out of ammunition. Thirdly, the fear that China and Pakistan would

16
CRITICAL REASONING

occupy positions of influence if India vacates them for too long. Fourthly, New
Delhi fears that India might lose its leverage to settle issues of bilateral interest such
as water, immigration control and trade, if it does not “engage” Gyanendra.
What attitude do you think India has towards Nepal?
(1) India has very friendly relation with Nepal. It wants to do the best to help
Nepal rid of Maoists.
(2) India has to help Nepal unwillingly because if it does not, China or Pakistan
can help Nepal and grow their influence in India’s neighbourhood.
(3) Maoists are a nuisance to India too. So, India wants strategic partnership
with Nepal and fight jointly against Maoists.
(4) India’s foreign policy is to maintain friendly relationship with its neighbours.
India’s help to Nepal is guided by this principle.
(7) As Pope Benedict XVI, Ratzinger may or may not prove his critics wrong.
Yet his pontificate is not going to be a carbon copy of that of his predecessor. There
will be “continuity” in so far as contentious and divisive issues are concerned -
Benedict XVI is unlikely to amend the Church’s teachings on women’s ordination,
homosexuality, clerical celibacy and so on. But it would have been the case, at least
in the foreseeable future, with any other person as Pope. Moreover, the legacy of
John Paul II will certainly have a significant bearing on this pontificate.
What do you assume is the opinions of critics about Pope Benedict XVI?
(1) Though liberal at heart, he has to bear the responsibility of the chair he is
holding; and so he has to continue the policies of the Church.
(2) John Paul II had been such a towering figure, so it is difficult for him to
override his principles.
(3) He may or may not be different from John Paul II but will more or less follow
his line.
(4) Joan Paul was rather more liberal than him. We are going to see the Church
more orthodox in days to come.
(8) If it is imperative that economic policy addresses the needs of nature and
society, and not just the market, can it do so without addressing the problem of
economic scale and redistribution and concern itself only with allocative efficiency
via right prices? Standard economics strains out the gnat of allocative efficiency
while swallowing the twin camels of unjust distribution and unsustainable scale. As
distribution becomes more unjust, big money buys political power and uses it to

17
ENGLISH USAGE

avoid any redistribution. A favourite ploy for avoiding redistribution is to emphasize


economic growth.
Based on the paragraph, which of the following inferences is true?
(1) Economic growth is just a ploy to divert public attention from redistribution.
(2) The concentration of money buys political power which results in further
unjust distribution.
(3) Right price of goods should be the prime goal of economic policy.
(4) Unjust distribution of wealth is the failure of market oriented economy.
(9) It is true that TADA’s failure to secure convictions stemmed from the fact
witnesses were afraid to testify. If intercepted communication could substitute
witness testimony, it would certainly constitute an improvement. If this provision
alone could help bring terrorists to justice, why was the government reluctant to
review other provisions of POTA that are seen as being draconian?
What does the content of the paragraph suggests about the author’s opinion
about POTA?
(1) The author thinks that POTA is a draconian law and questions the
government’s reluctance to change the oppressive provision.
(2) Like TADA, POTA will also fail because witnesses will fail to testify. So,
POTA may be necessay but it is not going to bear any result.
(3) With certain improvement, intercepted communication should substitute
witness testimony. Thus, the success of POTA can be secured.
(4) The provision that intercepted communication can substitute witness
testimony should be removed, as it is an infringement of individuals’
privacy.
(10) The Madras High Court has held that freedom of the press, which is
included in the fundamental right to freedom of speech and expression, “gives (the
press) a privilege in such matters even to exaggerate or play down to a small extent,
provided it is only in the interest of the public at large”. Justice A.K. Rajan made this
observation while dismissing a suit filed by Chemfab Alkalies Limited, Kalapet, near
Pondicherry, claiming damages of Rs.15 lakhs from Junior Vikatan, a Tamil biweekly,
for publishing what was described as a defamatory article. “So long as there is no
malice, it does not amount to libel,” the judge ruled.

18
CRITICAL REASONING

What does the judgement suggest ?


(1) Public interest is supreme, and the Press can distort a story if it is in the
interest of public.
(2) Society is more important than any organisation’s image. So, provided
society is benefited, a little harm to the organization does not matter much.
(3) The charge of defamation can be labeled only in the case of malice.
(4) The Constitution of India secures freedom of expression. Court has to
undertake the responsibility to uphold this right.

19
ENGLISH USAGE

EXERCISE – 2
DIRECTIONS for questions 1 to 18: Select the correct alternative from the given
choices.
1. The British approach to training was conditioned until recently by their belief
in the principles of ‘training by doing’ and ‘in learning from mistakes.’
Which of the following is not a situation for the application of the above principle?
(1) Learning how to solve a particular problem.
(2) Learning how to handle explosives by using actual explosives.
(3) Learning how to swim by swimming in shallow waters.
(4) Learning how to manage people by committing errors.
2. The advent of TCS is the time for genuine investors to exercise a discerning
judgement while choosing stocks, especially those of IT companies. Clearly
it is a congenial moment to reiterate what seasoned analysts knew all along
- it is the standing of the promoters that matters most.
Which, among the following, weakens the assertion made in the above
statements?
(1) The shares of some of the companies, not so well known, have not
done well despite performing well financially.
(2) The issue of a giant institution, National Thermal Power Corporation,
did well in the market.
(3) The equity shares of a leading commercial bank were over subscribed
several times.
(4) The shares of companies, big and small, belonging to I.T. have done
well.
3. Booker awards are not merely about who wins the coveted literary prize, an
annual honour that bestows instant celebrity status and a windfall in terms
of book sales. They are also about such things as who deserved to win,
why someone lost and whether someone else was unfairly overlooked -
issues that become the stuff of intense literary brawls, vicious backbiting
on occasion - but always engaging.
Which of the following, if true would seriously undermine the assessment
made in the above statements?
(1) The Nobel Prize for literature brings with it more acclaim, honour and
recognition than the Booker Prize.

20
CRITICAL REASONING

(2) Whenever any literary award is announced, it is usual for the people
in the field the world over to raise controversies, most of which are
unjustified.
(3) There have been some years, including the year in which Arundati
Roy was chosen for the award, when there had been universal approval
of the choices.
(4) This year’s Booker Prize for the novel ‘The line of beauty’ has raised
a controversy that it is not better than another entry ‘Cloud Atlas’.
4. No matter what people may profess to — ethics, values, beliefs, integrity —
money is the life, and idealism is but a glorified euphemism for the helplessness
and inability of man to get rich.
Which of the following statements best summarises the idea given in the
above paragraph?
(1) Some people do not try to get rich by devious means.
(2) Most of the people take a self-righteous approach only when it suits
them.
(3) People who do not profess ethics, values etc, do not worry if they are
not able to get rich.
(4) Money is not everything in life, people proper being happy than striving
for money.
5. There are a number of reasons as to why savers as a class need at least
policies to protect their returns if not new incentives to save. The resurgence
of inflation on the back of a strong and sustained increase in global oil
prices is one of them. With the inflation based on the wholesale price
index, for the week ended August 7, rising to within a grace of 8 percent,
few will deny that the most conventional forms of savings, such as bank
deposits, earn a negative real return.
Which among the following is consistent with what is stated in the above
paragraph?
(1) Inflation leads to a bull in the economic activities.
(2) Government must come forward with instruments of saving other than
bank deposits.
(3) Inflation causes erosion of value of money.
(4) There must be insurance policies to protect the people.

21
ENGLISH USAGE

6. A general purpose local network suitable for both terminal-to-computer


and computer-to-computer applications should be capable of meeting the
following quantitative requirements:
The ability to interconnect hundreds of terminals of various types of
computers, ranging from microcomputers to large systems, scattered
throughout a building or a collection of buildings. The ability to support
local-terminal to host communications with terminals operating upto at
least 19.2 kbps, including terminal virtualisation, by mapping characteristics
for various terminals and computers.
The ability to interface terminals to the network inexpensively (e.g. not more
than several hundred dollars per terminal).
The ability to support local computer-to-computer communication with
effective data rates in the order of several hundred thousand bits per second
or even a megabit per second.
Which of the following is true according to the paragraph?
(1) Local networks should be so organised, that it does maximum work in
a minimum period of time.
(2) Terminal-to-computer applications do not require local networking.
(3) The ability to interface terminals to the network inexpensively is one
of the requirements of a general purpose local network.
(4) Terminal virtualisation by mapping characteristics for various terminals
and computer is not one of the requirements of a local network.
7. The State Election Commission has decided to recommend to the
Government, enforcement of the two-child norm for Panchayat chief aspirants.
This is done as a measure to control population growth in the short term.
Which among the following, if true, makes the above reasoning absurd?
(1) In states where the norm has been in force, men have forced their
wives to abort the third child or give it away in adoption.
(2) Most men who currently occupy the post of Panchayat Chief are
unmarried.
(3) After the Cairo International Conference on population and
development, the focus has shifted from family planning measures
through punitive measures, to women and child welfare.

22
CRITICAL REASONING

(4) The aspirants for Panchayat chief posts are usually those who are past
their reproductive age.
8. The concept of the elasticity of demand is of great help in enabling a
government to fix a proper rate of foreign exchange for its currency that
will keep its balance of payments in equilibrium. At the time of taking
decisions to devalue or revalue the currency, the government should
carefully study the nature of the elasticity of demand and supply of its
exports as well as for its imports. If the government devalues the currency
without having made a careful study of the elasticity of demand for the
country’s exports and imports, it might fail in achieving its aim to correct
the imbalance in its balance of payments. Thus the concept of the elasticity
of demand is of great importance for a fuller understanding of many economic
problems.
Which of the following can be concluded from this paragraph?
(1) The concept of elasticity of demand is merely a mental exercise for
beginners.
(2) A country’s balance of payments can be kept in control by thoroughly
studying its elasticity of demand.
(3) The economy of a country least depends on its elasticity of demand.
(4) The elasticity coefficients do not vary when the scale of economic activity
changes.
9. The Commonwealth’s decision to suspend Pakistan indefinitely from its
grouping reflects the multinational organisations firm emphasis on
democratic and representative governance by its member-states. India,
through its diplomatic effort, lobbied actively for reprisal action against the
military regime in Pakistan.
Which of the following, if true, indicates that India is really not interested
in upholding democracy in Pakistan but it is only trying to meet some other
objectives of its own?
(1) India is contemplating snapping its diplomatic ties with Pakistan.
(2) India has not taken up any case against Pakistan’s military regime in fora
like the UN General Assembly or the UN Security Council.

23
ENGLISH USAGE

(3) India tried getting Pakistan suspended from the Commonwealth once
in the past also when the military took over power from the
democratically elected government.
(4) India wanted Pakistan to be censured by the Commonwealth if it could
not get Pakistan suspended.
10. The U.S. was most unhappy at the prospect of China acquiring nuclear
weapons. A year or so before China had its first nuclear test in 1964, some
sections in the U.S. Government considered the possibility of assisting
India to acquire nuclear weapons as a counterweight to China. It was only
after India and Pakistan conducted their tests of 1998 that sanctions were
imposed. The legal cover was the Nuclear Non Proliferation Treaty to which
neither India nor Pakistan are parties. Israel, also a non-signatory to the
NPT, was clandestinely helped by the U.S. to acquire nuclear weapons.
What can be inferred from the above paragraph?
(1) The U.S. is genuinely interested in nuclear non proliferation.
(2) The U.S. is selective in opposing other countries acquiring nuclear
capabilities.
(3) The U.S. treats China and India as its adversaries while it treats Pakistan
and Israel as its allies.
(4) The nuclear non proliferation treaty is against developing countries.
11. Though the Chinese position on India’s claim to a permanent seat in the U.N.
Security Council is still somewhat shrouded in ambiguity, Indian officials
feel that the trilateral agreement between India, Russia and China, to discuss
the high level report on U.N. reform, is a significant step in the direction of
making the U.N. Security Council more representative of current world realities.
It is universally accepted that India is a significant player on the world stage.
Which among the following, if true, would justify the optimism of Indian
officials about the proposed talks?
(l) The high level report on U.N. reform contains references to the
reconstitution of the U.N. Security Council taking into account present
global realities.

24
CRITICAL REASONING

(2) The high level report on U.N. reform recommends that prominent
developing countries be made permanent members of the U.N. Security
Council.
(3) Russia is a strong supporter of India becoming a permanent member of
the U.N. Security Council.
(4) China has not totally rejected India’s claim to a permanent seat in the U.N.
Security Council.
12. Ethionine, the ethyl analog of methionine, induces a variety of
pathological changes in experimental animals including fatty liver, acute
pancreatitis, and carcinoma of the liver. The acute effects on the liver
appear to be related to the induction of a fairly rapid and relatively large
decrease in the concentration of ATP which in turn is followed by an inhibition
of RNA and protein synthesis. In female rats, polysomes show the drop in
ATP and inhibition of protein synthesis in vivo, but the changes in polysomes
are minimal.
Which of the following can be concluded from this paragraph?
(1) The drop in the concentration of ATP and the inhibition of protein
synthesis in female rats least affects their polysomes.
(2) The effects of ethionine are completely reversed by the administration of
ATP precursors.
(3) The decrease in hepatic ATP concentration is the consequence of the
formation of SAE.
(4) Ethionine is responsible only for the changes that occur in the liver.
13. The on-going truck operators’ strike has built up the pressure on State
Transport Corporation buses and train services in the city.
Which of the following, if true, is the most plausible explanation of the
pressure referred to in the above paragraph?
(1) Hundreds of people who travel to the city on a regular bus prefer to
return home on trucks as the “fare” is affordable and lower than those
of buses and trains.
(2) Truck drivers, who are free because of the strike, are going on major
outings with their families.

25
ENGLISH USAGE

(3) Vegetables, milk etc that are usually transported by trucks are now
transported in trains and buses.
(4) Even on normal days, owing to the lack of sufficient number of State
Transport buses, people travel in private buses.
14. Russia’s incipient private sector suffered from many more disadvantages
than did the private sector in China. New firms are risky and it can therefore
be difficult for them to raise capital. Chinese firms had a tremendous
advantage in this regard due to the presence of an overseas network of
successful Chinese entrepreneurs, particularly in Hong Kong and Taiwan.
These individuals had the resources and the desire to establish a beachhead
in their homeland. As soon as China removed restrictions on foreign direct
investment, their money came pouring in. Russia, on the other hand, had to
go begging to the IMF, accepting all its conditionalities or to Western
multinationals, offering its heirlooms like oil, gas and other minerals in
exchange.
Which of the following can be a solution to this problem?
(1) Russia could approach China for some knowledge on how it manages or
controls its economy so well.
(2) The investments and aid from Western multinationals to Russia should
be minimised.
(3) The bureaucrats in Russia have to manage its economy better.
(4) Some Russian entrepreneurs who have established themselves abroad,
should volunteer to help the country as the Chinese entrepreneurs have
helped China.
15. Everything that we experience as material reality is born in an invisible
realm beyond space and time - a realm revealed by science to consist of energy
and information. This invisible source of all that exists is not an empty void but
the womb of creation itself. It turns the chaos of the quantum soup into stars,
galaxies, rainforests, human beings and our own thoughts, emotions, memories
and desires.
A logical continuation of this paragraph would be:
(1) It is possible not only to know this source of existence at an abstract level
but also to become intimate and be one with it.

26
CRITICAL REASONING

(2) There is a great stillness associated with this void.


(3) This void is vibrating silence.
(4) The energy present in the void is the creative force.
16. There is a price to pay for nuclear independence. Finance Minister Yashwant
Sinha is grappling with the problem of fixing the price and how to make the
nation pay it; how to make the world, the NRIs and everyone else come to
the aid of the party. We now know that there will not be global sanction,
only national hiccups. But with an economy on the slippery slope of
recession, even hiccups cause upheavals. So, as the people are in an upbeat
mood, the economy is on a slide. The Pokhran fallout may be politically
beneficial but economically disturbing.
Which of the following is true according to the paragraph?
(1) The nuclear test in Pokhran has proved harmful for the flow of foreign
investment into India.
(2) The India Development Fund has failed to meet the requirements of the
Indian economy.
(3) The Pokhran nuclear test has proved to be economically helpful, though
politically disturbing.
(4) Economy in India is on the slope of recession, and the Pokhran
experiment has added to its woes.
17. Intelligence is much more flexible than the mask of matter that hides it.
Intelligence can express itself either as thoughts or as molecules. A basic emotion
such as fear can be described as an abstract feeling or as a tangible molecule of the
hormone adrenaline.
From the passage it can be inferred that
(1) intelligence expresses itself as matter.
(2) intelligence expresses itself as sensations.
(3) intelligence can have both material and abstract manifestations.
(4) intelligence governs matter.

27
ENGLISH USAGE

18. The drop in the external parity of the rupee and speculation over the stability
of the government at the centre made the FIIs indulge in heavy liquidation
on the Bombay Stock Exchange. They have to also meet their commitments
in Hong Kong and other centres, where prices were on the downtrend.
Which of the following can be inferred from the above passage?
(1) When the FIIs liquidate their stocks, other investors also should do
the same.
(2) Stock markets across the world are completely unpredictable.
(3) Indian stock markets will not improve unless there is a stable
government at the centre.
(4) Indian stock markets depend not just on local conditions but also on
happenings outside the country.

28
2
Jumbled Paragraphs

In such type of questions, you are given a set of jumbled sentences and you
have to identify their logical sequence. Solving such questions is largely a matter of
common sense and your aptitude to reason; and it is not difficult to get a good hold
on them with regular practice. We will be dealing here with some key concepts that
will make your task considerably easier to crack such questions.
The most important task in handling these types of questions is to get a sense
of the structure of the statements. All arguments follow a sequence, which consists
of a beginning and an end, and the two are joined by ‘links’. So, your job is to…
1. Identify the beginning or the opening sentence.
or/and
2. Identify the end or the concluding sentence.
or/and-
3. Identify the link.
And on this basis, you have to eliminate the wrong choices. With some suitable
examples, let us see how we can work out this prescription.
Example 1 :
A. They spent over Rs. 120 crore a day through credit cards during the year.
B. Indians were sold 45,000 credit cards a day in 2004.
C. But they tell only half of the story.
D. Both the figures represent more than 60% rise over the previous year.
(1) ABDC (2) BADC (3) BCDA (4) DACB
Solution: In this example we have more than one clue. In sentence D ‘both the
figures’ obviously refer to the figures in A and B. So, D has to come after A and B. In
choices 3 and 4, D is not preceded by A and B. So, we are left with choices 1 and 2.
Now, between A and B, A will come after B, because the pronoun ‘they’ in A is
coming for the plural noun ‘Indians’ in B. Obviously, B is the opening sentence. So,
the answer is 2]
ENGLISH USAGE

To be remembered
An opening sentence can have a noun whose corresponding pronoun you
can find in latter sentence(s). It is a vital clue for spotting the sequence of the
sentences.
Example 2 :
A. The art form began in UK in 1950s and moved to the US in the ’60s.
B. Film memorabilia constitutes one part of pop art.
C. Popular art takes its subject matter, techniques from everyday world.
D. Artists such as Andy Warhol, Roy Lichtenstein and Claes Oldenburg
exemplify this style.
(1) CADB (2) ADCB (3) BCDA (4) DCAB
Solution : In this example, we see that the subject is ‘popular art’. A can not be the
opening sentence because the definite article ‘the’ is used before ‘art’ which suggests
that the subject has been already introduced. B is telling you only about ‘one part
of pop art’. In D ‘this style’ refers to the style which is already mentioned. In C
‘popular art’ is mentioned without any qualification. Only in 1] C is the opening
sentence. So, the answer is 1]

To be remembered
In English usage, subject is generally introduced without the definite article. It
can appear without any article if the subject is plural or uncountable, and with
an indefinite article (a, an) if the subject is singular.
Example 3 :
A. The flare can be seen as bright, almost white area in the EIT 195.
B. A good-sized X-ray flare and a CME erupted from the sun from the area of
Active Region.
C. Because the sunspot was just about facing directly towards the earth, the
particle cloud rushed out of the sun heading towards the earth.
D. Scientists call this phenomenon as “EIT wave”.
(1) ABCD (2) BDAC (3) DCBA (4) BADC

30
JUMBLED PARAGRAPHS

Solution : Problems like this can easily be solved by tracking the links. It is clear
here that the opening sentence is B. The subject ‘X-ray flare’ is introduced with the
indefinite article ‘a’ (as suggested in Example 2). The word ‘flare’ provides the link
to the sentence A. Again ‘EIT’ in A supplies link to D. The sequence has to be
BADC. So, the answer is 4].

To be remembered
It is not safe to depend solely on the word link. Sometimes it can be misleading.
It is better to take into account the sequential development of the theme also.
One should not just find the link and drop to the conclusion.
Example 4 :
A. Man generally identifies himself with his body-mind-intellect personality
without knowing his true nature, which is divine.
B. Physical body, thoughts and emotions keep changing all the time and
hence the human personality that is obvious to experience fails this test.
C. It is the wrong identification, which is the root of his sorrows.
D. The spiritual quest, which every individual undertakes whether consciously
or unconsciously, is the urge to know his essential nature.
(1) ABDC (2) DACB (3) C BDA (4) BACD
Solution : In this example, there appear two probable openings D and A. Both
option 1] and 2], then, can be probable answers. Now, we have to apply different
method here. After first glance we can find that C is a natural consquence of A. The
‘wrong identification’ in C is about the ‘body-mind-intellect’ identification in A.
We find that in option 1] does not have AC squence. So, the answer is 2]

To be remembered
We see in the example above that the opening is not quite clear. Two statements
appear to begin the argument. In such cases, you should go ahead with probable
options, and then see further which of them follows the logical sequence, as
we have done in this example.

31
ENGLISH USAGE

Example 5 :
A. In the Bhagavad Gita, Lord Krishna offers Arjuna the opportunity to
overcome the tension by becoming a practitioner of Karma-Yoga
B. The tension of the yogis has been between gaining worldly experience
and living as a renouncer.
C. Thus one remains active in the world without renouncing the world.
D. Here one endeavors to perform actions in accordance with one’s inherent
talents without having attachment to the fruits.
(1) DBAC (2) DCBA (3) BADC (4) ABCD
Solution : In the given example, it is difficult to identify the beginning, as A and B
both claim the opening. However, the concluding sentence is apparent. Anyone can
see that the argument is ending with C. Now, option 1] and 3] both are ending with
C. But 1] begins with D which does not appear to be the opening sentence. Now, we
are left with only 3]. So, the answer is 3]

To be remembered
In a question, where the opening sentence is not clear, as in this, the identification
of the concluding sentence can give a vital clue. If you find that there is only
one option ending with the sentence you have identified, the answer is clear to
you. For example, if we had only one choice ending with C in the above example,
it would have been the answer. But if there are two or more such options, you
have to toil a bit and look for the probable beginning or the links.

32
JUMBLED PARAGRAPHS

EXERCISES – 3
1 A. The prospects of millions of men forced to go solo threatens major social
and political problem.
B. China has paid the price for a rapid decline in fertility combined with a
preference for male children.
C. China is becoming the world’s largest lonely heart club with some 23 million
men of marriageable age unable to find a female partner.
D. From the year 2000 and continuing until 2020, there will be many boys of
marriageable age seeking females to marry who will be unsuccessful in
their courtship pursuits.
(1) BDCA (2) CABD (3) DBAC (4) CADB
2. A. Friedman, who is one of the regular columnists for The New York Times
could not contain himself.
B. The French and the Dutch and Europe in general had rebuked the
globalization that Friedman cherishes.
C. Shortly after Friedman’s book appeared, the French and the Dutch rejected
the European Union’s draft constitution
D. “French voters are trying to preserve a 35-hour work week in a world where
Indian engineers are ready to work a 35-hour day”, Friedman said.
(1) BCDA (2) CBAD (3) CABD (4) ACDB
3. A. Maunima herself was married at the age of five.
B. “What is wrong with this?” said Mauniama.
C. “Widowed or divorced women marrying again is taboo”, she said.
D. Many of the marriages end in divorce and only men can marry again.
(1) BCDA (2) DACB (3) DBCA (4) BCAD
4. A. The report suggests that the High Court direct the central government to
frame policies that would support marginal farmers.
B. The report holds that long-term measures should be the rehabilitation of a
system, in this case the agrarian production system itself.
C. The court should make the Union government party to the present
situation.
D. Policies must support cultivation so that farmers stay out of the debt trap.
(1) BADC (2) BCAD (3) ACDB (4) DABC

33
ENGLISH USAGE

5. A. Marshal Bulganin said it was first necessary for the two Germanys – East
and West – to make contact in order to render later unification possible.
B. Sir Anthony repeated his thesis that Germany should be united through
free elections with possible demilitarization of a special eastern zone.
C. The British Prime Minister, Sir Anthony Eden, and Marshal Bulgania gave
their point on the unification of Germany.
D. The big four leaders got down to business at their summit conference in
Geneva on July 19 to ease East-West tension on an agenda headed by the
problem of German Unity.
(1) DACB (2) DBAC (3) DABC (4) DCBA
6. A. Gold buying witnesses a steep rise in festive seasons.
B. To attract customers, jewellers make many attracting offers those days.
C. Gold has always been the biggest attraction for Indian consumers.
D. Buying gold is also investing.
(1) ABCD (2) DCBA (3) CDAB (4) BACD
7. A. Easter is about the belief of triumph of life over death.
B. To hope for life after death is a tendency of human nature.
C. What if anything awaits beyond the grave?
D. Since our first thinking ancestors, the mystery of death has haunted
humankind.
(1) ADCB (2) CADB (3) BDAC (4) DBCA
8. A. Tribal art is close to nature and natural events.
B. This has resulted in some of most wonderful works of art.
C. Rituals play a major role in it and it is closely linked to food, security, health
and survival.
D. The lack of codified rules makes it colourful, vibrant and evocative, the
natural expression of people.
(1) ACDB (2) ACBD (3) DBAC (4) BDCA

34
JUMBLED PARAGRAPHS

9. A. The psychic task, which a person can and must set for himself, is not to
feel secure but to be able to tolerate insecurity without panic and being
under fear.
B. When we make a decision, we are never certain of its outcome; any decision
implies a risk of failure.
C. We can never be certain of the outcome of our decision of our best efforts,
the result always depends on many factors that transcend our capacity to
control.
D. Because of the very condition of our existence we can not feel secure
about anything.
(1) CDAB (2) DABC (3) BCDA (4) ABCD
10. A. At 12 weeks, babies usually use both hands with equal vigour.
B. When the in infant reaches the age of 2 years, the right hand takes over
again, but between 2 ½ and 3 ½, bilateral activities begin again.
C. By 16 weeks, they mostly favour the left hand while making contact.
D. At 32 weeks, they are again bilateral.
(1) DCBA (2) BADC (3) ACBD (4) ACDB
11. A. Persuasion is a manifestation of force which we use in everyday life.
B. We do not tell them, ‘many people think the earth is round, and a few think
it is flat. But we tell them, ‘earth is round’.
C. Many forms of persuasion- even many of which everybody approves- are
really a kind of force.
D. Consider what we do to our children.
(1) DABC (2) BCAD (3) CBDA (4) ADBC
12. A. The divorce rate is found to be higher among working couples.
B. Children with working mothers, said a recent survey, enjoy higher level of
self-confidence and assertiveness.
C. Working women are unable to do justice to either to their home or to their
career.
D. Men with working wives tend to work harder at their jobs.
(1) BDCA (2) DCAB (3) BACD (4) ABDC

35
ENGLISH USAGE

13. A. Pliny’s nephew, known as Pliny the Younger, was with him on that day, but
he stayed back at Misenum.
B. A foolhardy Pliny the Elder launched ships and sailed towards the erupting
volcano for a closer look only to be suffocated to death.
C. At the time of the eruption of Mt. Vesuvius, the Roman fleet under the
command of Pliny, the Elder was stationed across the Bay of Naples
D . This is an excerpt from a live account of the events of that fateful day,
recorded for posterity by Pliny the Younger, a Roman historian.
(1) ABDC (2) DCBA (3) BCDA (4) DCAB
14. A. They predict that the U.S., Russia and France would vie with one another
to sell light water reactors to India
B. Officials of the Department of Atomic Energy have had no hesitation in
saying that Bush offers would boost India’s nuclear power capacity.
C. India could also become a ‘global player’ in the nuclear market.
D. Besides India too could sell its Pressurized Heavy Water Reactors to several
countries that are keen on buying them.
(1) ABCD (2) ABDC (3) BACD (4) BADC
15. A. Yet another order of January 23, 2004, by SA, Agra, appointed Sadiq Ali the
Imam of Taj Mahal masjid on a salary of Rs.15 per month.
B. Considering the huge revenues from the Taj Mahal, this is a pittance of a
salary for a functionary working in the complex.
C. On October 16, 2003, the Superintending Archeologist, Agra, opened the
gates of Taj Mahal for the special namaz of Taraveeh in Ramzan.
D. On September 9, 2004 under UPA government, the ASI gave permission for
the three-day annual urs of Shahjahan to be held at the Taj Mahal.
(1) ABCD (2) DCBA (3) BACD (4) CDAB

36
JUMBLED PARAGRAPHS

EXERCISE – 4
DIRECTIONS for questions 1 to 3: A number of sentences are given below which,
when properly sequenced, form a coherent paragraph. Each sentence is labelled
with a letter. Choose the most logical order of sentences, from among the four given
choices, to construct a coherent paragraph.
1. A. The interactive multimedia-training concept is seen by trainers and
educators as a “natural” way to learn.
B. This facility for individual involvement is believed to enhance
knowledge absorption and retention rate.
C. Multimedia is going to play a major role in effective training in the near
future.
D. This is because it provides the user with an experience involving sight,
sound and in some cases, touch.
(1) CDAB (2) CDBA (3) CADB (4) CBAD
2. A. A cut in the interest rates is high on the agenda of expectations of
both banks and companies for the credit policy for the busy season.
B. They also expect a relaxation of the 5 percent limit of incremental
deposits that can be invested in corporate shares and debentures.
C. There are also expectations that the number of interest rate slabs will
be reduced to two from the present three.
D. Sections of the market expect a relaxation in the lending norms against
shares.
(1) ABCD (2) ABDC (3) DBAC (4) ACDB
3. A. “This is legalisation of quackery,” exclaims the president of the
Indian Medical Association.
B. Not only that, you will be allowed to prescribe medicines from a list of
drugs used for common ailments and administer first aid.
C. And with a certificate of 10 years experience in providing medical
assistance, signed by a qualified doctor or a sarpanch of the relevant
area, and by paying a sum of Rs.4,500, you will be issued a certificate
declaring you as a “health volunteer”.
D. All you need to be is a matriculate with science subjects.
(1) ABCD (2) DCBA (3) DBAC (4) BCAD

37
ENGLISH USAGE

DIRECTIONS for questions 4 to 7: In each of the following questions,


statements 1 to 6 are respectively the first and last sentences of a paragraph.
Statements A, B, C and D come in between them. Rearrange A, B, C and D in such
a manner that they make a coherent paragraph together with statements 1 and 6.
Select the correct order from the given choices and mark its number as your
answer.
4. 1. Let a man then know his worth, and keep things under his feet.
A. But the man in the street, finding no worth in himself which corresponds
to the force which built a tower or sculptured a marble God, feels poor
when he looks at these.
B. Yet they all are his, suitors for his notice, petitioners to his faculties
that they will come out and take possession.
C. To him a palace, a statue, or a costly book have an alien and forbidding
air, much like a gay equipage, and seem to say, “Who are you, Sir?”
D. Let him not peep or steal, or skulk up and down with the air of a
charity-boy, a bastard, or an interloper in the world which exists for him.
6. The picture waits for his verdict; it is not to command him, but he is to
settle its claim to praise.
(1) BDAC (2) DBAC (3) BCDA (4) DACB
5. 1. Scientists believe that tree-kangaroos are a product of a peculiar
observation of natural history known as ping-pong evolution.
A. As the continent dried and the forests retreated, its descendants adapted
to life on the ground, diversifying into an array of terrestrial wallabies,
kangaroos, over 70 species of which are known to exist today.
B. Though it retained the tree-kangaroo’s powerful hind legs, it eventually
became a good enough climber to live permanently on trees.
C. The ancestor of all kangaroos was a cat-sized tree-dweller that lived
more than 24 million years ago when much of Australia was cloaked in
rainforest.
D. Fossils suggest that about four-and-a-half million years ago, a rock
wallaby-like creature began to spend more and more time clambering up
sloping tree trunks.
6. Then it diversified quickly, adapting to many different forest habitats.
(1) BCDA (2) DBCA (3) ADCB (4) CADB

38
JUMBLED PARAGRAPHS

6. 1. The mistake that many earnest and persevering talkers make is to suppose
that to be engrossed is the same thing as being engrossing.
A. People who are fond of talking ought to beware of being lengthy.
B. It is true of conversation, as of many other things, that the half is better
than the whole.
C. How one knows the despair of conversing with a man who is determined
to make a clear and complete statement of everything, and not to let his
hearer off anything.
D. Arguments, questions, views, rise in the mind in the course of the
harangue, and are swept away by the moving stream.
6. Such talkers suffer from a complacent feeling that their information is
correct and complete, and their deductions are necessarily sound.
(1) DABC (2) BACD (3) ACDB (4) CBDA
7. 1. Bengal which used to be considered as a nursery of Indian football,
draws a blank now.
A. At present, plenty of foreign players adorn the ‘big’ clubs like Mohun
Bagan, East Bengal and even Tollygunge Agragami.
B. Even the key matches of the National League played in Calcutta like
‘East Bengal-Churchill Brothers’ fail to draw crowds.
C. Conditions are almost the same in other parts of the country.
D. Young children and their parents have lost interest in football, as cricket
has soaked in all the glamour.
6. Veteran football coaches have some suggestions to offer to rejuvenate
Indian football.
(1) ADBC (2) BCDA (3) CBDA (4) DBCA
DIRECTIONS for questions 8 to 11: Each question below has four or five sentences
followed by four suggested ways of arranging the sentences. Among the four
ways, three of them give a logically meaningful and coherent paragraph whereas
one does not. Identify the choice that does not give a coherent paragraph and mark
that as your answer.
8. A. The Indian market came down by 10% and it is not clear whether this is just
the beginning of another phase.

39
ENGLISH USAGE

B. The benchmark Bombay Stock Exchange Sensitive Index (Sensex)


crashed by 368 points to 2830 from 3198 recorded the previous weekend.
C. The tremors following the crash of the World Trade Centre towers in
New York were felt globally, particularly in the financial markets.
D. On the National Stock Exchange (NSE), the S & P CNX Nifty dipped by
115 points to 92, from the previous Friday’s close of 1036.
E. At present, India is suffering from recession and the problems in the
U.S. only added to its woes.
(1) CADBE (2) CABDE (3) CDABE (4) CEADB
9. A. Man is a social animal and he cannot live in isolation.
B. Friendship is an indispensable factor in human life.
C. So, contact with one another is a natural urge in man.
D. He also likes to hear from others.
E. Moreover, a man finds relief when he unloads his heart to others.
(1) BACDE (2) BACED (3) ABCDE (4) ABCED
10. A. Success means the fast integration of today’s new technologies and
preparation now for the next generation of tools tomorrow.
B. At the same time, most individuals struggle to keep up with technology
they already have, and are unable to grasp the full impact of the future.
C. Today’s technology is tomorrow’s dinosaur.
D. They stumble from one new software application to the next, a new
e-mail system, a word processor upgrade, a new telephone switchboard,
a new manufacturing process.
E. The trouble is that most companies are obsessed with pushing accepted
technology to the limits, when the greatest long-term threat is probably
going to come from a technology which is new and different so different
that very few can take it seriously today.
(1) AEDBC (2) CEBDA (3) CAEDB (4) ACBED
11. A. As a result, Derrida was adulated by feminists, gay activists and other
marginalized groups which applied his theories to show up discrimination
and racial and sexist biases in accepted texts and ideas.

40
JUMBLED PARAGRAPHS

B. The Cornerstone of Derrida’s work, continued to be the theory of


deconstruction through which he attempted to show that language is
constantly shifting and that a text has multiple legitimate interpretations.
C. This led to a rereading of texts by Shakespeare or the Greek philosophers
which unravelled hidden biases and contradictions.
D. His concept found wide acceptance, particularly in the United States, and
was applied to a broad range of subjects including literary theory, linguistics,
art, music and political science.
E. There could therefore be no absolutes in terms of truth, meaning or permanence.
(1) BDEAC (2) BCDAE (3) BEDCA (4) BADEC
DIRECTIONS for questions 12 to 15: Arrange the statements A, B, C and D between
statements 1 and 6 to make a logically coherent paragraph.
12. 1. Many beauty-related problems are caused by stress, poor health, and
low self-esteem and therefore can only be solved by adopting a holistic
approach.
A. For instance, even though we live in the microsecond, blink-and-
you’ve-missed-it computer era, there is no such thing as an instant beauty
solution.
B. The main problem with quick-fix solutions is that they do no such
thing as endowing beauty instantly.
C. There are, however, many beauty related myths and stereotypes, which
are counterproductive to the pursuit of bettering ourselves.
D. Anyone and anything that promises ‘dramatic overnight results’ should
be approached with scepticism.
6. There is no substitute for perseverance, discipline and a slow and
steady approach.
(1) ADCB (2) CABD (3) CDBA (4) ABCD
13. 1. Hitler was reluctant to engage in war with the British, not because he
was particularly concerned with British military powers (he was not)
but because he regarded the British as second only to the Germans in
their ‘Aryan’ purity.

41
ENGLISH USAGE

A. Yet the diversity of people living in Britan then and now needed to be
transended through what is referred to as a collective leap of the
imagination.
B. But to say that something is imagined does not mean that it is imaginary.
C. In effect, because the English or any other population hardly know
each other, they have to imagine the similarities that apparently bind
them together.
D. But identity is imagined, for to be English - as Hilter usually referred to
the British – implies having a whole raft of things in common with every
other English person.
6. Identity is constructed not discovered; it is a product of the imagination
rather than a product of history, let alone science.
(1) DACB (2) ABCD (3) CBAD (4) DBAC
14. 1. What is driving energy consumption growth?
A. Today, oi l’s pr imacy as the preferred t ransportation fuel is
unchallenged.
B. Gas, the celebrated clear competitor to oil, is generally viewed as a
regional resource whose economics work best when transported through
pipelines within the region.
C. No large scale viable alternative to oil is as yet in sight when it comes to
driving cars, trucks, trains, ships and planes.
D. For most developing countries, the transportation sector accounts for
more than half the oil consumption.
6. The assessment holds, despite the emerging markets for its expensive
sibling, the liquefied version.
(l) DABC (2) ABCD (3) DACB (4) ACBD
15. 1. For the common man the one clear message from the monetary policy
is that the era of historically low interest rates has come to an end.
A. Naturally, retail borrowers (home loans and other forms of retail lending
have almost wholly lifted the aggregate credit off-take) will have to pay
more.
B. The two goals may conflict with each other.

42
JUMBLED PARAGRAPHS

C. The RBI has sharply raised the inflation target and says that
maintenance of price stability will be a prime goal, ranking on a par with
its other objective of providing liquidity for credit and investment needs.
D. For if in the process of targeting inflations the RBI were to signal a higher
interest rate or take steps to contract liquidity, the cost and volume of
credit will be affected.
6. They may also be faced with a situation where the lenders will be less
forthcoming in pushing their loan products.
(1) DCBA (2) ACBD (3) CBDA (4) CABD
DIRECTIONS for questions 16 to 18: Each question below has multiple sentences
followed by four suggested ways of arranging the sentences. Among the four
ways, three of them give a logically meaningful and coherent paragraph whereas
one does not. Identify the choice that does not give a coherent paragraph and mark
that as your answer.
16. A. The immediate use of the technology would be in powering missiles or
military aircraft, initially pilotless or remote-controlled.
B. The X-43A aircraft flew at an altitude of 1,11,000 feet at the record speed
for just 90 seconds, but left behind a load of data for scientists to study
and analyse.
C. This could dramatically change the complexion of future air warfare
where speed would determine the cutting edge.
D. Dubbed ‘90 seconds of terror’, the X-43A test flight is just a first step
in nudging flying speed to sky high limits.
E. The brief show of speed in the sky over the Californian coast when a
pilotless plane flew at the velocity of sound, which translates to about
11,300 km an hour, marks a new milestone in science, particularly in
aviation technology.
F. Now researchers will have to find how this achievement can be
translated into use - either for military or civilian purposes.
(1) EBDFAC (2) EFBDAC (3) DCBEFA (4) BCDEFA
17. A. In Rio, the rich countries tended to talk about deforestation and the
destruction of the tropical forests.
B. The poorer countries of the south, meanwhile emphasised wasteful
consumption in the Western world.
43
ENGLISH USAGE

C. At the Rio de Janeiro “Earth Summit” two years ago, the population
issue was barely mentioned.
D. The focus there was on the linkage between the environment and
development.
(1) CADB (2) CDBA (3) CABD (4) CDAB
18. A. He seemed the favourite to become the UN’s Secretary General in 1981,
but the Soviet Union vetoed his candidature, claiming he was too pro-
western, and vetoed him again in 1991.
B. It was almost certainly nonsense, but the Russians may have believed it.
C. Around that time, stories circulated that die prince was a secret agent
for the British, using his job as a cover for intelligence gathering.
D. Sadruddin stepped down after 12 years, the longest any refugee chief
has held the job, but returned to the UN at times of crisis to give help,
notably in Afghanistan, during the Soviet occupation and in Iraq, after
the first Gulf War.
E. Sadruddin insisted that he had equal sympathy for eastern and western
people.
(1) DACBE (2) DCAEB (3) EACDB (4) BCADE
DIRECTIONS for questions 19 to 20: Arrange the statements A, B, C and D between
statements 1 and 6 to make a logically coherent paragraph.
19. 1. Probability is explicitly central to scientific methodology in so far as it
is crucial to the foundation of statistics.
A. Statistical methods are also employed in the discovery of causal
relationships.
B. The proportion intending to do so, is ascertained in some sample and
then statistical methods are used to establish how reliable an indicator
this is of the proportion intending to do so in the entire population.
C. Statistical methods are exploited as a shortcut to the determination of
the frequency with which some attribute occurs in a proportion too large
to examine in its entirety.
D. If we wish to know what proportion of the UK electorate intends to
vote ‘Labour’, we won’t have the resources to question each elector
individually.

44
JUMBLED PARAGRAPHS

6. To this end, they are again applied to finite bodies of data, such as a
specification of the incidence of lung cancer in two groups of men in
their fifth decade.
(1) CDAB (2) CDBA (3) DCAB (4) DABC
20. 1. Education has been defined as the technique of transmitting civilisation
and it is shocking that the country with the oldest and greatest
civilisation should be so lackadaisical about the technique of
transmitting it.
A. More criminals have openly entered public life than ever before and
no democracy can last long in such circumstances.
B. Without guidance which can be derived only from liberal education, a
whole generation has grown up, which is content to see crime and
violence, casteism and communal frenzy become the order of the day.
C. The Indian psyche today, remains wholly untouched by any thought of
the need for wider and more value-based education.
D. Education has never been a high-priority item in any Indian political party’s
manifesto, so the subject which should have galvanized the nation into
action fifty-five years ago is still kept in cold storage.
6. It is now acknowledged all over the world that value-based education
is the only instrument for transmuting national talent into national
progress.
(1) BCAD (2) ADBC (3) DACB (4) CDBA

45
3
Fill in the Blanks
In Fill in the Blanks, you are required to pick up the most suitable word or
phrase from the options and fill it in the blank space of the sentence in question.
Your selection of the option should be such that the sentence should appear plausible
and give out a logical meaning.
The first step towards reaching the right option is to read the sentence carefully
and try to get a sense of its meaning. Of course, the meaning is not complete. It will
be complete only when the right option is filled in the blank space. The tone/style of
the passage is the most crucial factor in judging the options. Knowledge about
positive/negative nuances amongst the words is very helpful. You have to judge
which option will make the sentence sound logically complete.
Fill in the Blanks are generally of five types.
1. Logical Fill in the Blanks.
2. Paired Logical Fill in the Blanks.
3. Missing Links in the Paragraph.
4. Theme completion.
5. Maximum number of times.
We will see how such questions can be tackled with the help of some examples.

1. LOGICAL FILL IN THE BLANKS


Example 1 : The dispute over Dalits’ participation in the annual car festival of Sri
Swarnamoorthi Eswarar temple at Kandadevi has been …… brought to the Madras
High Court over the past seven years.
(1) unknowingly (2) repeatedly (3) vehemently (4) frustratingly
Solution : All the options are grammatically correct but not all options are corre
sponding to the tone of the sentence. The phrase ‘over the past seven year’ suggests
that the action has been carried out with regularity. So, no other option is better
suited than ‘repeatedly’. The answer is 2]
FILL IN THE BLANKS

Example 2 : Building on the strong…foundation laid by its predecessors, the


entrepreneur generation of Coimbatore turns the industrial city into an advanced
center of education in the country.
(1) entrepreneur (2) industrial (3) acaooodemic (4) agricultural
Solution : Option 4] is ruled out, as there is no hint of agriculture. The rest three
options could be confusing. But notice that the city is turning into a center of
‘education’. So, the foundation on which it is being built on has to relate to education.
So, the option 3] is the answer.
Example 3 : Can anyone guarantee a job or a better life after the children…school?
There are no answers to the question.
(1) complete (2) join (3) leave (4) like
Solution : To ‘join’ and ‘like’ school, of course, do not show possibility of a job. The
words ‘complete’ and ‘leave’ are synonymous in one context. But children can leave
the school without completing the education. Only ‘complete’indicates that the
education is finished which can augment the possibility of a better life. So, the
answer is 1]
Example 4 : There is widespread…the implementation of the welfare programme
because a large number of tribal families have not been rehabilitated so far.
(1) assurance about (2) faith in
(3) awareness of (4) scepticism about
Solution : The sentence has a negative tone, so option 1] and 2] do not fit in
because of their positive aspect. The two clauses are joined by ‘because’ which
shows cause and effect relation between them. ‘Awareness’ can not be a result of
the lack of rehabilitation. People are skeptical about the success of the new
programme because the earlier work is still incomplete. So, the answer is 4]
Example 5 : The project to carve the world’s largest Buddha statue, five times
bigger than the Bumiyan, is evidently very…
(1) annoying (2) ambitious (3) futile (4) necessary
Solution : A grand project could be ‘annoying’, ‘necessary’ and even ‘futile’ but the
statement does not give any indication to reach such conclusions. Whereas, to
build the biggest statue of Buddha in the world is unmistakably ambitious. So, the
answer is 2]

47
ENGLISH USAGE

EXERCISE – 5
1. Jinnah very dexterously used religion, because Islam has historically provided
a rallying point for the…of Muslim community.
(1) enlightenment (2) stagnation (3) contentment (4) mobilization
2. The younger, smarter crop of models is successfully fashioning alternative…
for themselves, while still walking on the ramps.
(1) careers (2) amusements (3) knowledge (4) disciplines
3. By babbling in competitive religiosity, the ruling Congress and the Akalis have
…separatism in Punjab.
(1) forbidden (2) restricted (3) celebrated (4) encouraged
4. Besides enhancing the economic growth the scheme will also open many ……
avenues.
(1) entertainment (2) education (3) leisure (4) employment
5. Greed is a desire to obtain more money or material … or bodily satisfaction than
one is considered to need.
(1) renunciation (2) accumulation (3) salvation (4) identification

2. PAIRED LOGICAL FILL IN THE BLANKS


There are two blanks in Paired Logical Fill in the Blanks questions. This can
be easier than the first type as it has more clues to offer. Normally, the two blanks will
be part of two ideas that is propounded in the passage. The connecter between the
i9deas is a dead give away. It may denote contrast, support, cause-effect etc. Hence,
erroneous option can be ruled out easily. Tone of the passage is still important in
these type of questions.
Example 6 : What is… is almost always the least… and the most attractive may
even be harmful.
(1) important…beautiful (2) useful…attractive
(3) useful…ugly (4) attractive…harmful
Solution : There are two propositions here. In the latter one we come to know that
‘the most attractive’ can be ‘harmful’. The first proposition should be setting a
contrast to the second one; that is, ‘useful’ can be ‘ least attractive’. So, the answer
is 2]

48
FILL IN THE BLANKS

Example 7 : The …of nature is he whose inward and outward senses are still truly
…to each other.
(1) lover….adjusted (2) lover…antagonistic
(3) knower…known (4) hater…adjusted
Solution : Love is a positive feeling. The ‘lover’ of nature can’t have his inner and
outer senses ‘antagonistic’ to each other. On the other hand, someone with
harmonized senses cannot be a ‘hater’. Option 3] does not appear pertinent. So the
answer is 1]
Example 8 : With globalization, the…of Indian economy can no longer be… in
isolation.
(1) nuances…understood (2) future…assessed
(3) progress…known (4) damage…done
Solution : A common understanding tells us that options 1] and 2] are more suitable
options that the other two. Globalization is not an essential condition to damage or
know the progress of Indian economy. We can rule out 3] and 4]. On further
deliberation, one can deduce that 2] is more probable. The future of Indian economy
is unavoidably dependant on outer factors in this era of globalization.
Example 9 : The…in draught afflicted Sahel in Africa is showing a rather…pattern.
(1) rainfall…unusual (2) people…unusual
(3) animals…usual (4) rainfall…usual
Solution : Draught is not a normal condition. It is generally caused due to the lack
of rainfall. Obviously, rainfall won’t be showing usual pattern if there is draught. So,
the answer is 1]
Example 10 : Without much …it can be argued that urban education in India could
well be ……… with the best anywhere in the world
(1) pleasure…compared (2) sense…paralleled
(3) knowledge…labeled (4) hesitation …compared
Solution : The options for the second blank space have more or less the same
meaning. So, let us focus on the first. If Indian education stands among the best in
the world, it is a matter of pride and confidence. It would be best, thus, if the
sentence starts with, ‘without much hesitation’. So, the answer is 4]

49
ENGLISH USAGE

EXERCISE – 6
1. Direction and pace of economic…is frequently…by the availability of resources,
adequacy of food, transport facilities, technical how-know etc.
(1) recess…accelerated (2) boom…determined
(3) development…hampered (4) development…determined
2. The…of vehicles have recorded a steep…notwithstanding the skyrocketing
rise in prices of vehicles and fuel.
(1) sales….increase (2) misuse…increase
(3) sales…decrease (4) use…decrease.
3. I am not attracted by the......life of the......, always wandering through the country-
side, begging for charity.
(1) proud...almsgiver (2) affluent.....mendicant
(3) peripatetic....vagabound (4) noble....philanthropist
4. If the Titanic had hit the icerberg head on, its water-tight compartments might
have saved it from......, but it sverved avoid the iceberg, and in the collision so
many compartments were opened in the sea that disaster was......
(1) foundering....inevitable (2) sinking....escaped
(3) buoyancy....unavoidable (4) damage....limited
5. It is foolish to vent your epleen on......object; still, you make......enemises that
way.
(1) an inspied....dull (2) a humane....more
(3) an interesting....curious (4) an inanimate....fewer

3. MISSING LINK IN PARAGRAPH


You are given a whole paragraph in this type of question. There are some
missing words here and there, which you have to fill with one of the choices given
at the end of the paragraph. Your first step should be to read the paragraph carefully
and try to take out a rough gist of it. Take into account what is the central issue and
other peripheral issues around it. It makes your work fairly easy if you are able to
have an understanding of the theme.
50
FILL IN THE BLANKS

After reading it, you need to go over the blanks, and see which of the choices
fits comfortably in the context of the paragraph and helps to advance the argument.
Keep in mind that the options may be confusing but there must be one and only one
right answer.
Following are some examples with explanatory solutions
Example 1 : Of all its accomplishments, the West is perhaps most 11 of its scientific
revolution, which has been unfolding for the past half-millennium. In the centuries
preceding this intellectual sea change, the Arab world played a pivotal historical
role. Its own scholars 12 nature and pushed the bounds of knowledge, while its
scribes preserved the discoveries and insights of earlier thinkers whose works did
not fit well with the prevalent Christian dogma of a world unfolding according to a
divinely predetermined plan. In recent times, however, Arab and Muslim societies
have turned away from science, 13 these societies from enjoying its many benefits.
Wasim Maziak argues that all of us now should have a vested interest in advancing
science and technology in the Arab and Muslim world. Not only can science and
technology help to feed people, improve their health, and create wealth, but they
can help reduce societal 14 and build international bridges for badly needed dialogue
and mutual understanding. To usher science and technology more thoroughly into
Arab culture and society, however, Maziak suggests that the West needs to 15 the
Arab world’s historical contributions, and he advises the Arab world to stop dwelling
on its golden past by embracing lessons about science and technology that the
West learned long ago.
11. (1) ashamed (2) proud (3) scared (4) fond
12. (1) loved (2) revered (3) studied (4) suspected
13. (1) precluding (2) encouraging (3) helping (4) benefiting
14. (1) tensions (2) prosperity (3) obligations (4) customs
15. (1) slander (2) eulogize (3) acknowledge (4) overlook

SOLUTION
11. If you read a couple of lines further, you will know that science has been dealt
positively. So, there is no need of negative choices 1] and 3]. On second
examination, ‘fond’ seems not quite appropriate here when you have option
‘proud’. So, the answer is 2]

51
ENGLISH USAGE

12. In the same sentence, you have phrase, ‘pushed the bound of knowledge’.
Understandably, it can be accomplished with studies. So, the answer is 3]
13. We are well aware of the fallout of turning away from science and what it has
done to the Arab world. So, the answer is 1]
14. You can argue that more one choices can be used here. But the tone of the
sentence and the paragraphs is far from elaborating the ills of science. So, the
answer is 1]
15. Among all the given paragraphs, it is to ‘acknowledge’ Arab’s contribution that
can boost scientific development again there. So, the answer is 3]
Example 2 : The ability to identify genetic signs of diseases is advancing faster
than the ability to treat those diseases. For Mayana Zatz, a professor of genetics at
São Paulo University in Brazil, this 16 is a compelling call to conduct basic laboratory
research into such diseases and to do what she can to improve the lives of those 17
by them. In the case of Duchenne muscular dystrophy and other types of muscular
dystrophy, Zatz has helped 18 the biomolecular bases of some of these diseases,
counselled thousands of affected individuals and families about how to manage
their condition, and established an association and clinic to help improve the lives
of her country’s poorest victims of Duchenne muscular dystrophy. Zatz chronicles
the research, ethical 19 and legal challenges that she 20 in her molecules-to-society
attack on Duchenne muscular dystrophy and other neuromuscular disorders.
16. (1) harmony (2) misfortune (3) nonsense (4) mismatch
17. (1) profited (2) afflicted (3) saddened (4) pleased
18. (1) uncover (2) shroud (3) spread (4) eliminate
19. (1) conundrums (2) solutions (3) people (4) groups
20. (1) helps (2) understands (3) confronts (4) likes

SOLUTION
16. The ability to identify the diseases is more than the ability to treat them. This is
certainly a ‘mismatch’. So, the answer is 4]
17. Unless one is a sadist, one is not going to be ‘pleased’ by diseases. One could
best be ‘afflicted’ by them. So, the answer is 2]
18. Options 2 and 3 are ruled out in the first glance. Had he eliminated them, he
would not have counseled people ‘how to manage’ their condition, as eliminated
things need not be managed. So, the answer is 1]
52
FILL IN THE BLANKS

19. Given the context of the sentence containing the phrase ‘legal challenges’ it is
evident that the choice ‘conundrums’ will be suitable as it is synonymous with
‘challenges’. So, the answer is 1]
20. Challenges are confronted. So, the answer is 3]
Example 3 : QFA offers the security and the environment generally 21 by the
international business community. Such environment takes the form of incentives
consisting of availability of natural gas at very competitive prices, a wide range of
tax concessions and 22 financial and currency arrangement. The Free Area’s laws
formally contemplate provision of a tax holiday by the authority and unrestricted
circulation and convertibility of any amount of foreign currency and capital. The 23
also formally prohibit all forms of confiscation and expropriation of property rights
and provide for the establishment of a mutually 24 court or an independent forum
possessing the characteristics expected of an international arbitration tribunal to
conside 25 and trade disputes arising between the FREE AREA’S licensees
21. (1) discarded (2) unnoticed (3) sought (4) jeered at
22. (1) unessential (2) beneficial (3) harmless (4) harmful
23. (1) regulations (2) prohibitions (3) authorities (4) limitations
24. (1) disregarded (2) disagreed (3) agreed (4) understood
25. (1) financial (2) legal (3) authorial (4) academic

SOLUTION
21. The security is sought, and so it is offered. So, the answer is 3]
22. The services of QFA are beneficial. So, the answer is 2]
23. In the previous sentence the subject is ‘laws’. As the same subject is carried in
this sentence too, then ‘regulations’ which is a synonym of ‘laws’ will be the
right choice. So, the answer is 1]
24. Among all options, ‘mutually’ should be followed by ‘agreed’. So, the answer
is 3]
25. With ‘trade’, ‘financial’ is the best-suited option. So, the option is 1]
Example 4 : When William Shakespeare took a 26 280 molecules out of every
million entering his lungs were carbon dioxide. Each time you 27 breath today,
380 molecules per million are carbon dioxide. That portion climbs about two molecules

53
ENGLISH USAGE

every year. No one knows the exact consequences of this 28 in the atmosphere’s
carbon dioxide (CO2) concentration nor the effects that lie ahead as more and more
of the gas enters the air in the coming decades—humankind is running an uncontrolled
experiment on the world. Scientists know that carbon dioxide is warming the
atmosphere, which in turn is causing sea level to rise, and that the CO2 absorbed by
the oceans is 29 the water. But they are unsure of exactly how climate could 30
across the globe, how fast sea level might rise, what a more acidic ocean could
mean, which ecological systems on land and in the sea would be most vulnerable to
climate change and how these developments might affect human health and well-
being.
26. (1) bath (2) nap (3) breath (4) sandwich
27. (1) draw (2) hold (3) puff out (4) release
28. (1) upsurge (2) decline (3) constancy (4) desertion
29. (1) acidifying (2) salting (3) soiling (4) hardening
30. (1) be destroyed (2) be constant (3) improve (4) alter

SOLUTION
26. Carbon dioxide enters lungs through ‘breath’. So, the answer is 3]
27. Of course, draw. So, the answer is 1]
28. Again it is evident that carbon dioxide is increasing in the atmosphere. So, the
answer is 1]
29. The effect of CO2 in water is to make it poisonous. So, the answer is 1]
30. There could be two possible choices, 1] and 4]. But the option 1] is not supported
by the sentence. So, the answer is 4]
Example 5 : Deaths caused by smoking cost the United States about $92 billion in
lost productivity between 1997-2001, according to a new report from the Centers for
Disease Control and Prevention. This amount is up nearly $10 billion from the
estimates for 1995-1999. Combined with smoking-related health care expenses, that
figure jumps to $167 billion per year. The report says that on average, smoking 31
life expectancy by approximately 14 years. Smoking, which can cause 32 diseases,
lung cancer, and other health problems, is responsible for 440,000 deaths each year
in the United States. A 2001 Institute of Medicine report examined pharmaceutical

54
FILL IN THE BLANKS

and modified tobacco products designed to reduce the health risks of smoking and
found that these items cannot yet be proved to 33 tobacco-related disease. Clearing
the Smoke: Assessing the Science Base for Tobacco Harm Reduction ( 2001 )
outlines how tried and true public health tools — research, surveillance,
communication, and regulation — should be used to ensure that the availability of
these products confers 34 risk to the individual and to the population as a whole
compared with conventional tobacco products. It 35 a regulatory strategy to prevent
cigarettes with greater toxicity than those sold today from entering the market and
to gather complete information about new harm reduction products.
31. (1) boosts (2) reduces (3) deteriorate (4) does not boost
32. (1) heart (2) allergic (3) respiratory (4) contagious
33. (1) reduce (2) intensify (3) cure (4) transfer
34. (1) more (2) less (3) no (4) too much
35. (1) forbids (2) mocks (3) recommends (4) dislikes

SOLUTION
31. It does need an elaboration that smoking has a negative effect on health. We
can exclude 1] and 4] then. The verb ‘deteriorate’ is used in qualitative terms
and so, won’t be appropriate with ‘life expectancy. So, the answer is 2]
32. Smoking directly affects respiratory system. So, the answer is 3]
33. This section of the paragraph tells us about the ‘modified tobacco’ which is
supposed to reduce the health risk. The word ‘yet’ in the sentence suggests
that the examination could not prove what was expected of the product. So, the
answer is 1]
34. ‘These products’ in the sentence refers to the ‘modified tobacco’. They are
expected to be less harmful than the products made of common tobacco.
Consequently, the risk is less too. So, the answer is 2]
35. The pronoun ‘it’ in the sentence stands for ‘Institute of Medicine’. The motive
here is to ‘prevent cigarettes with greater toxicity’. Clearly, a ‘regulatory strategy’
should be recommended. So, the answer is 3]

55
ENGLISH USAGE

EXERCISE – 7
DIRECTIONS for questions 1 to 25: In the following passage there are blanks, each
of which has been numbered. These numbers are printed below the passage and
against each, four words are suggested, one of which fits the blanks appropriately.
Find the appropriate word in each case.
1. Neo-classical economics begins with the premise that resources are 1… and
that it is necessary to choose between competing alternatives. That is, economics
deals with trade-offs. With scarcity, choosing one alternative implies forgoing
another alternative – the opportunity cost. The opportunity costs creates an
implicit price relationship between competing alternatives. In addition, in both
market oriented and 2… economies, scarcity is often explicitly quantified by
price relationships. Understanding choices by individuals and groups is central.
Economists believe that incentives and desires play 3… role in shaping decision
making. Concepts from the Utilitarian school of philosophy are used as analytical
concepts within economics, though economists appreciate that society may
not adopt utilitarian objectives. One example of this is the idea of a utility
function, which is assumed to represent how economic agents rank the choices
given to them. A given 4… alternative can be thought of as a vector where the
entries are answers to questions like “How many eggs should I buy?”, “How
many hours should I spend with my kids?”, and “How much money should I
set aside for later?”. Then the utility function ranks these from best to worst,
and the agent gradually learns to 5… the best-ranked choice in the feasible set
of his alternatives.
1. (1) abundant (2) limitless (3) scarce (4) important
2. (1) free (2) planned (3) small (4) macro
3. (1) no (2) little (3) a negative (4) an important
4. (1) good (2) profitable (3) economic (4) well planned
5. (1) loathe (2) select (3) like (4) applaude
2. Archetype is defined as the original model of which all other similar persons,
objects, or concepts are merely derivative, copied, patterned, or emulated. The
term is often used in literature, architecture, and the arts to refer to something
that goes back to the fundamental 6 of style, method, gold standard, or physical
construct. Shakespeare, for example, is epitomized for 7 many archetypal
characters, not because he was the first that we know of to write them, but
56
FILL IN THE BLANKS

because he defined those roles amongst the backdrop of a complex, social 8


landscape. Thus, the characters stand out as 9 by contrast, even though many
of his characters were based on previously-garnered archetypes (Shakespeare
often borrowed from fables, myths and magic to construct and embellish his
plays).The imitation process of an archetype or prototype itself is called a
pastiche (noun), in which one who 10 pays homage to the original creator.
6. (1) ideas (2) origins (3) understanding (4) theories
7. (1) popularising (2) ravishing (3) having (4) reading
8. (1) scientific (2) humanitarian (3) philosophic (4) literary
9. (1) boring (2) duplicate (3) original (4) lively
10. (1) sings (2) mimics (3) bafoons (4) mourns
3. A distinction is often drawn between the “natural” and the “artificial” (“man-
made”). Can such a distinction be justified? One approach is to exclude mind
from the realm of the natural; another is to exclude not only mind, but also
humans and their influence. In either case, the boundary between the natural
and the artificial is a 11 one to draw. Some people believe that the problem is
best 12 by saying that everything is natural, but that does little to clarify the
concept of the “artificial”. In any event, 13about the distinction between the
natural and the artificial animate much of art, literature and philosophy. Another
approach is to distinguish natural processes and artificial (man-made) processes.
In this viewpoint, a process is deemed to occur either at the behest of man, or
not. For example, flipping a light switch might illuminate a room, or perhaps a
sunrise might illuminate that room. In this viewpoint, the sunrise would be
termed a natural process; the decision of a human being to flip the light switch
would be termed an artificial illumination, in contrast. In this viewpoint, artifice
(art or literature) is clearly the result of willful 14 action; furthermore, the act of
stating a philosophical position is also a willful action (and hence at the behest
of man), whether or not the content of the 15 were to be about science.
11. (1) very easy (2) not difficult (3) difficult (4) easy
12. (1) solved (2) avoided (3) understood (4) accepted
13. (1) clarity (2) obviousness (3) simplicity (4) ambiguity
14. (1) divine (2) human (3) absurd (4) natural
15. (1) philosophy (2) action (3) willfullness (4) science

57
ENGLISH USAGE

4. Philosophers typically frame problems in a 16 manner, historically using


syllogisms of traditional logic, since Frege and Russell increasingly using formal
systems, such as predicate calculus, and then work towards a solution based
on critical reading and 17 Like Socrates, they search for answers through
discussion, responding to the arguments of others, or careful personal 18.
Philosophers often debate the relative merits of these methods. For example,
they may ask whether philosophical “solutions” are objective, definitive, and
say something informative about reality. On the other hand, they may ask
whether these solutions give greater clarity or insight into the logic of language,
or rather act as personal therapy. Philosophers seek justification for the answers
to their questions. Language is the philosopher’s primary tool. In the analytic
tradition, debates about philosophical method have been closely connected to
debates about the relationship between philosophy and 19. There is a similar
concern in continental philosophy. Meta-philosophy, the “philosophy of
philosophy”, studies the nature of philosophical problems and solutions, and
the proper method for getting from one to another. These debates are also
connected to debates over language and interpretation. These debates are not
less relevant to philosophy as a whole, since the nature and role of philosophy
itself has always been an essential part of philosophical 20.
16. (1) interesting (2) ridiculuous (3) rational (4) tiring
17. (1) reasoning (2) bantar (3) remarks (4) response
18. (1) criticism (2) contemplation (3) debate (4) ejoyment
19. (1) science (2) politics (3) religion (4) language
20. (1) gatherings (2) sages (3) deliberations (4)squabble
5. In the 19th century, the Westward expansion of the United States incrementally
21 large numbers of Native Americans from vast areas of their territory, either
by forcing them into marginal lands farther and farther west, or by outright
massacres. Under President Andrew Jackson, Congress passed the Indian
Removal Act of 1830, which forced the Five Civilized Tribes from the 22
reservations, primarily to take their land for settlement. The forced migration
was marked by great hardship and many deaths. Its route is known as the Trail
of Tears. Conflicts generally known at the time as “Indian Wars” broke out
between 23 forces and many different tribes. Authorities entered numerous
treaties during this period, but later 24 many for various reasons. Well-known

58
FILL IN THE BLANKS

military engagements include the atypical Native American victory at the Battle
of Little Bighorn in 1876, and the 25 of Native Americans at Wounded Knee in
1890. On January 31, 1876 the United States government ordered all remaining
Native Americans to move into reservations or reserves. This, together with
the near extinction of the American Bison which many tribes had lived on, set
about the downturn of Prairie Culture that had developed around the use of the
horse for hunting, travel and trading.
21. (1) invited (2) expelled (3) honoured (4) took away
22. (1) east to western (2) west to eastern
(3) north to southern (4) south to northern
23. (1) UK (2) US (3) Native American (4) Latin American
24. (1) sustained (2) made (3) abrogated (4) entertained
25. (1) massacre (2) entertainment (3) generosity (4) kindness

4. COMPLETE THE THEME


In such questions you are given 3, 4 sentences in an order with one sentence
missing. The missing sentence could be in the middle or at the end. You have to
complete the theme by filling up the missing sentence from the options given below.
You have to employ you logical sense here. You need to read the sentences carefully.
There would be a break in the theme due to the missing sentence. The missing
sentence would provide the continuation of the theme and make it complete. The
sentences before and after the missing sentence can give you the hint.
Example 36 : In the middle of this commotion, he spotted an impressive policeman
who was trying to direct the traffic.
He fought his way over to him to inquire where the bazaar was.
_________________________________________
This bridge was the only link between the twin city of Kolkata and Howrah
1. The bazaar was a filthy, noisy place.
2. The bridge was one of the biggest he had seen in his life.
3. The policeman growled that it lay on the other side of the bridge
4. The policeman got so angry that he asked him to jump into the river from
the bridge.

59
ENGLISH USAGE

Solution : After reading the sentence before the missing link, one would expect
what the policeman told the man. Option 1] and 2] tell you about the ‘bazaar’ and the
‘bridge’ but they sound incomplete as there is no mention of the policeman’s reply.
4] does not sound reasonable as there is no mention of why the policeman should
get so angry. So, the answer is 3]
Example 37 : The WHO had stipulated a certain number of tests to be undertaken
before transfusions of blood.
They made possible the detection of hepatitis B virus or any venereal diseases.
Yet at the hospital the virus seemed the last worry.
_________________________________________
1. There were no proper equipments for the test.
2. All that really mattered was profit.
3. There had never been any case of polluted blood there.
4. The pressure of patients made them forget about it.
Solution : Notice, ‘the virus seemed the last worry’. It clearly points out negligence
on the part of the hospital. Options 1] and 4] show helplessness of the hospital but
they do not paint it in negatively. Whereas, 3] shows complacence of hospital
staffs. 2] proves their culpability. They are negligent because they are concerned
just about profit. So, the answer is 2]
Example 38 : The role of women in Indian villages is a thankless one.They are up
before everyone else and last to go to bed. Throughout the day they work like
slaves.
_________________________________________
1. No domestic task, no drudgery is considered too much for them.
2. They are too modest too receive thanks without embarrassment.
3. The men will not acknowledge their toil.
4. The lack of education makes them prey to all kinds of oppression.
Solution : After a fleeting glance, options 1] 3] and 4], all appear probable answers
as they all can hold grounds for bad condition of women in Indian villages. But after
paying more attention you can see that the sentences in the question are describing
the hardships of women in Indian villages. They do not tell you why the condition
of women is such as options 3] and 4] are doing. In other words, they are explaining
‘how’ like option 1]. Options 3] and 4] seems to be answering ‘why’. The nature of

60
FILL IN THE BLANKS

1] is similar to that of other sentences in the question. So, the answer is 1]


Example 39 : A few hundred Venezuelan doctors paraded on the streets of Caracas
to protest against the Cuban doctors and para-medical staff working in their country.
_________________________________________
Besides this economic insecurity, the other allegation is that the real goal of the
Cuban doctors and other professionals is “political subversion”.
1. The medical training in Cuba is not adequate.
2. They maintain that Cuban doctors are depriving qualified Venezuelan
doctors from earning a livelihood.
3. Cuba has a communist regime, and so the doctors from that country may
not have only economic and humanitarian motives.
4. Being foreigners, they don’t have sympathetic attitude towards the patients
of Venezuela.
Solution : The last sentence in the question mentions, ‘economic insecurity’. This
is an important clue. It means the preceding sentence mentions something about it.
Options 2] and 3] relate to economic issues. But 3] does not give hint of economic
‘insecurity’. Rather it is more about, let us say, political insecurity. 2] talks of depriving
jobs of Venezuelan doctors by the Cuban one. The economic insecurity is apparent
here. So, the answer is 2]
Example 40 : The most pronounced impact of terrorism on children is psychological.
_________________________________________
Similarly, when children are forced to move to an unfamiliar world, the psychological
and social impact on them can be shattering.
This phenomenon is equally applicable where the child has to move to another
family or place after the death of either of his/her parents.
1. Scientists have been working on it, and have recently brought out this
stunning revelation.
2. Children’s minds are more receptive to whatever is going on around them.
3. Children are the future of our country, and they need to be preserved from
exposure to violence.
4. In a society where children continue to witness violence, they suffer from
various psychosomatic and psychiatric ailments.

61
ENGLISH USAGE

Solution : In the question, the sentence after the missing link starts with ‘similarly’,
hinting that the disposition of the preceding sentence should be alike. After a
careful attention you will find that among all the options, option 4] has a similar
character and the sentence after the missing link seems to be building on it. So,
the answer is 4].

EXERCISE – 8
1. The collapse of the Japanese imperial economy at the end of the World War 2
and the colossal human losses led to a distinctive economic structure.
_________________________________________
This relatively less ideological and planned approach to economic resurgence
accounts for Japan’s impression as, “a market-oriented society”, trying to
privatise its postal services so late in the day.
More significantly, the idea of political control over the cash-rich postal services
has had a magical effect on Japan’s domestic scene.
1. The radically new economic policies were chalked out and implemented
with much deliberation.
2. The service industry suffered a huge downfall as there was an acute dearth
of human resource.
3. The post-imperial Japanese leaders wanted to put their country’s economy
back on course by whatever means possible.
4. The postal service should have been privatized even after WW2 as it was
incurring lose then.
2. In the gruesome Gurgaon incident in which many workers were severely beaten
by the police, the Honda management was the real enemy.
The policies of the central government under the liberalization programme afford
scope for companies to show losses, cease workers’ salaries etc. at their will
_________________________________________
There will be more Gurgaon like battles in the future.
1. The management gurus find loopholes in the company laws and the open
economic policy.
2. The workers should have refrained from resorting to violence.

62
FILL IN THE BLANKS

3. The Honda management did not give bonus to the workers which it had
promised.
4. There should be better arrangement of law and order in the places prone to
such commotion.
3. The cult of sacred space often involves a ritual separation from the site from its
profane surroundings.
Gentiles were barred from the Jewish temples.
_________________________________________
Under the Christian Byzantines, Jews had never been allowed to reside
permanently in the city.
1. Moses had ordered the Jews not to let non-Jews enter their temples.
2. Non-Muslims are still forbidden to enter Mecca.
3. Jews took revenge of their humiliation since ancient age.
4. Jews, Christians and Muslims are intolerant of one another.
4. Despite possessing the purchasing power, Indian consumer lack the maturity
to buy software from legal sources.
In developed nations, companies make extra effort of creating awareness about
the development stages and resources of software products.
_________________________________________
In return, consumers are more than willing to pay for the value of the product.
1. It is mainly because a majority of Indians have seen a sudden flood of
software in the country but have no clue of its origin.
2. The lack of professional attitude among Indians is the cause of it.
3. Per capita income is high in the developed countries, and people there can
afford to buy original software.
4. Companies do not make effort to create awareness in India as they do in
developed countries.
5. Carnatic Sangeet, a South Indian system of music, is found in states of Tamil
Nadu, Kerala, Andhra Pradesh and Karnataka.
These states are known for their strong presentation of Dravidian culture.
_________________________________________

63
ENGLISH USAGE

Whenever Westerners think of Indian music, they immediately think of Ravi


Shankar and the sitar.
1. These states are rich in other art forms as well.
2. Dravidian culture dates back to the ancient time.
3. In the West, Carnatic Sangeet is not as well known as north Indian music
4. The West has hardly any sense of the varieties of Indian music.

5. MAXIMUM NUMBER OF TIMES


DIRECTIONS for questions: Each question below has a set four sentences A, B, C
and D with one blank each. There are four words following these sentences. Each of
the words can fit into one or more of the four sentences. Find the maximum number
of times any word can fit into the blanks given in each set of sentences and mark
that as your answer.
Example 41 :
1. Modern societies created hegemonic categories of science verses magic,
technology verses…
2. It is often said that there is a great difference between… and religion.
3. …is still rampant in those societies which are still untouched by modernity.
4. There is a very thin line between… and faith.
(1) stereotype (2) superstition (3) science (4) superiority
Solution : After reading the sentences carefully in example, you can see that option
1] can come in sentence 3, as stereotyping is an irrational act. There can be a
contrast between science and religion. So, option 3] can come in 2. But option 2] can
be placed comfortable in all the four sentences. So, the answer is 2]
Example 42 :
1. Long term investors have a clearly laid out plan that will allow them to
reach their long term … objectives
2. The cause of migration from villages to cities is mainly…
3. All superstructures in a society religion, morality, art etc are dependant on
the …relations the individuals of the society live in.
4. In today’s world, relation between two countries rests on pragmatism and
mutual … considerations.
(1) social (2) economic (3) political (4) humanitarian

64
FILL IN THE BLANKS

Solution : There could be ‘social’ reason of migration. Relation between two countries
can rest on ‘political’ or ‘humanitarian’ considerations. But ‘economic’ reason is
responsible for all the activities mentioned in all the four sentences. So, the answer
is 2]
Example 43 :
1. A good way to counter … to change is to preempt the possibility of
occurring.
2. … is reduced when people feel that they have had the opportunity to
participate and express their views.
3. The French Revolution was a great example of … against the oppressive
feudal political system.
4. When the police surrounded the house, the kidnappers showed no … and
surrendered.
(1) obstacle (2) courage (3) anxiety (4) resistance
Solution : In sentence 1.‘obstacle’ can be filled in. Again ‘courage’ can fill in the
blanks of 3 and 4. But ‘resistance’ suits in all the four sentences. So, the answer is 4]
Example 44 :
1. If the idealized lifestyle is unavailable to all …, then on what basis do a few
countries hope to maintain such a lifestyle?
2. … suffered at the hand of Hitler.
3. Scarcities are mainly man-made or the earth has enough to feed and
shelter the whole …
4. If we start practicing the Gandhian philosophy of love and non-violence,
the whole … will be in the better condition.
(1) animals (2) mankind (3) Jews (4) Africa
Solution : It is only too obvious that the word ‘mankind’ can fit in all the blank
spaces. So the answer is 2]
Example 45 :
1. A society with degraded … can not be healthy.
2. Aboriginal … is an all encompassing spiritual path and way of living,
based on complex relationships between people, spirit ancestors, animals
and the land.

65
ENGLISH USAGE

3. The paranoiacs are so much enamoured with their own … that they don’t
see any merit in others.
4. The international demand of Indian cinema has raised the interest in Indian
… abroad.
(1) culture (2) society (3) people (4) manners
Solution : The word ‘people’ can come in 1 and 4. Again, ‘society’ can occur in 2 and
4. ‘Manners’ can be suitable in 14. But ‘culture’ can come in all the places. So, the
answer is 1]

EXERCISE – 9
DIRECTIONS for questions: Each question below has a set four sentences A, B, C
and D with one blank each. There are four words following these sentences. Each of
the words can fit into one or more of the four sentences. Find the maximum number
of times any word can fit into the blanks given in each set of sentences and mark
that as your answer.
1. 1. In both Islamic and Biblical traditions, the … world is described in relation
to the world that one will enter in the life hereafter.
2. A realist believes that the … conditions of a society are responsible for
shaping its individuals.
3. The west, though much advanced in … comforts, still looks towards the
East for spiritual upliftment.
4. An ascetic is not much concerned with the … world.
(1) virtuous (2) economic (3) material (4) sinful
2. 1. … is the reality of life.
2. I met him after ten years and found him with no …
3. Due to liberalization, Indian metropolitan society has witnessed a great …
in the last 10 years.
4. I was in too much hurry to notice any … in her attitude.
(1) death (2) improvement (3) haughtiness (4) change
3. 1. They company’s global … is to gradually shift the export base from South
Korea to India so as to reduce shipping costs.
2. The adopted a clever ….to win the final match.
66
FILL IN THE BLANKS

3. Chennai is an important base for the chemical ...


4. There are indications that the Iraqis have formed a broad … to drive out
the occupation forces.
(1) front (2) industry (3) strategy (4) technique
4. 1. The rapid growth in the company’s profit is the result of the … of the
workers.
2. Huge human resource has given Indian leading role in the field of service
….....
3. The new finance minister has laid emphasis on the development of rural
....… for overall GDP growth.
4. The one big disadvantage of the arrival the MNCs is the adverse impact
on small scale ….....
(1) industry (2) sector (3) agriculture (4) resources
5. 1. Man is the product of heredity and ….....
2. The preservation of … is the underlying thought behind sustainable
development.
3. The seminar mainly discussed the fallout of big dams on … and people
living in the surrounding areas.
4. Many companies flagrantly violate the regulations on …. in order to earn
more and more profit.
(1) plants (2) environment (3) animals (4) water

EXERCISE – 10
DIRECTIONS for questions 1 to 4: Each question has a sentence with two blanks
followed by four pairs of words. From the choices, select the pair of words that best
completes the given sentence.
1. According to Gandhi, the politician and the social worker must, like the
scientist, conduct experiments with utmost accuracy, forethought and
minuteness, and never claim _______ or make any subtle pretention to
_________.
(1) invisibility...eminence (2) victory...sovereignty
(3) finality...infallibility (4) sainthood...depravity

67
ENGLISH USAGE

2. The ______ association of women with emotionality and men with emotional
restraint bears some historical relation to an association of women with the
private, domestic and _______ roles and men with public, employment and
ascendant roles.
(1) autarchic...supine (2) stereotypical...subservient
(3) mistaken...salutary (4) misplaced...ambivalent
3. The Punjab Assembly’s Bill ____ all previous agreements and accords on
river waters and the assent given to it at ____speed by the state governor
raise many serious issues.
(1) annulling . . . ponderous (2) terminating ... lightning
(3) testifying ... breakneck (4) challenging ... visible
4. Given the initial ______ the view that metaphorical uses of language are
insufficiently precise to be scientific, it is surprising that the use of metaphors in
science is so______.
(1) plausibility of. . . pervasive (2) uproar on ... blatant
(3) paradox that. . . fallacious (4) skepticism about. . . risible
DIRECTIONS for questions 5 to 8: Each question below has a paragraph given
with one sentence missing in between. From among the answer choices given,
select the sentence that can fill the blank to form a coherent paragraph.
5. __________, but also meet the rising expectations of Indian workers and the
needs of the developing Indian economy.
(1) Not only will linking wages to productivity help reward honest and sincere
workmen
(2) Linking wages to productivity will help reward not only honest and sincere
workmen
(3) Linking wages to productivity will not only help reward honest and sincere
workmen
(4) Linking wages to productivity rewarded not only honest and sincere
workmen
6. Modern wars are supposed to be short affairs, but the struggle in former
Yugoslavia has, from the start, had a medieval quality, conveyed both by
the antique origins of the quarrel and by the low- tech means with which it
68
FILL IN THE BLANKS

is pursued. ——. The ultimate victor in Bosnia may turn out to be the side
with the most durable will to win.
(1) It is unlike any other war fought in the twentieth century.
(2) The modern wars, like the defeat of Iraq by the U.S., are what can be
called “wang-bang-slam” affairs.
(3) Above all, it is a war of siege and strangulation.
(4) Till now, there is no clear victor from among the Croats, the Muslims and
the Serbs.
7 The entire tragedy ____the Catholics and the Protestants.
(1) arose because of the hatred developed by
(2) flowed from the mutual hatred harboured by
(3) unfolded because of the unseen hatred between
(4) developed on the premise of mutual hatred between
8. Governments serious about economic development will ____of free
expression, and support the same, not withstanding any likely political fallout.
(1) discourage the economic power
(2) counter the far-reaching effects
(3) support the economic implications
(4) recognise the economic significance
DIRECTIONS for questions 9 to 11: Each question below has a set four sentences
A, B, C and D with one blank each. There are four words following these sentences.
Each of the words can fit into one or more of the four sentences. Find the maximum
number of times any word can fit into the blanks given in each set of sentences and
mark that as your answer.
For example, if a word can fit into the blanks in 2 of the four sentences and if
that is the maximum number of times any word can fit into the blanks in the set
of sentences then the answer for that question is 2.
9. A. The _______ was legally binding on both the parties.
B. A/An _____ was finally reached between the hostile neighbours.
C. These substances _______ on cooling.
D. The exercise helps to ______ abdominal muscles.
(1) expand (2) agreement (3) contract (4) tighten

69
ENGLISH USAGE

10. A. The lawyer _____the witness closely to verify his statements.


B. The government refused to ____ with the hijackers.
C. Renu found it difficult to ____the sharp curves on the ghat road,
D. The doctor her ____carefully but could find nothing wrong with her.
(1) inspected (2) negotiate (3) discuss (4) examined
11. A. The government has made strike illegal in __________ services.
B. Though it does not guarantee happiness, money is _________ to lead a
happy life.
C. Being loquacious is _______ to succeed as a compere.
D. A good dictionary is ______ to learn a foreign language.
(1) essential (2) vital (3) necessary (4) dispensable
DIRECTIONS for questions 12 to 16: Each question has a sentence with two
blanks, followed by four pairs of words as choices. From the choices, select the pair
of words that can best complete the given sentence.
12. Even as domestic insurers blithely ______on their commitments and
governments turned a blind eye, _____insurance and reinsurance companies
took the losses on their chins.
(1) fulfilled . . . audacious (2) reneged . .. prosperous
(3) defaulted . . . foreign (4) repudiated . .. international
13. Arguing the prior existence of an Indian nation was _______ to the
nationalist struggle against colonialism and it was a claim made by
ideologically _______ nationalists, to contest the attempts by the colonial
masters to emphasize India’s manifold social divisions.
(1) extrinsic . . . diverse (2) intrinsic . . . disparate
(3) inherent. . . homogeneous (4) innate . . . inhibited
14. We must allow the ____to come from everywhere, to freshen us and to teach
us, but if we allow them to ______us away, then we would have no roots in our
country.
(1) suggestion ... carry (2) criticism ... abrade
(3) winds . . . blow (4) knowledge ... cast
15. The latest amendments to the Patents Act 1970, expected to be introduced in
Parliament to _____ with India’s commitments under the ‘Trade-Related

70
FILL IN THE BLANKS

Intellectual Property Rights’ agreement of the WTO have ____almost


unanimous criticism.
(1) concur ... invited (2) compare ... solicited
(3) comply ... invoked (4) conform ... induced
16. Gandhi’s repeated triumphs against human________, stunned his
awestruck countrymen and made him an object of reverence, even to those
who were otherwise very _______ of his fasts and religious appeals.
(1) barbarism . . . appreciative (2) frivolity . . . dismissive
(3) savagery . . . critical (4) benignity . . . deprecatory
DIRECTIONS for questions 17 to 20: Each statement has a part missing. Choose
the best option from the options given below the statement to make up the
missing part.
17. As I was observing two minutes silence in the midst of tribal families who
are victims of starvation even while 60 million tonnes of grain are rotting in the
stores________.
(1) I could not think of economic policy as a form of terrorism pushing
people into poverty and starvation
(2) I could not help thinking of economic policies pushing people into
poverty and starvation is a form of terrorism
(3) I could not help but think of economic policies which push people into
poverty and starvation as a form of terrorism
(4) I could not help but think what pushes people into poverty and
starvation is economic policy as a form of terrorism
18. If the introduction of a purchase tax on listed securities has set the cat among
the pigeons, the lack of clarity over its application to mutual funds has _____.
(1) compounded the problem
(2) led to tax collection on a phenomenal scale
(3) caused untold misery to the common man
(4) bled the common man at the altar of tax collection
19. The Indian Railways had initially evolved as an integrated set-up consisting
of distinct disciplines of allied specialities. The set-up for maintenance and

71
ENGLISH USAGE

operation of infrastructure as well as commercial activities got organised


as one entity. ——. The available resources were shared between different
activities in a balanced measure.
(1) However, this made the Railways a monolith incapable of responding
to the needs of the passengers.
(2) Because of this, both the areas suffered badly.
(3) This enabled the Railways to work with lesser budgets.
(4) This gave better co-ordination of the activities and an integrated approach.
20. The Indian national congress underwent the distinct transformation;
_______ and, quite naturally, its agenda was to capture and retain political
power at the center as well as in the states.
(1) far from being a platform of nationalism to a political party
(2) of being a platform of nationalism in a political party
(3) from being a platform of nationalism to a political party
(4) as being a platform of nationalism in a political party

72
4
Syllogism
Syllogism is a form of reasoning in which a conclusion is drawn from two given
or assumed propositions (premises).
Syllogisms can be best understood with the help of VENN DIAGRAMS
VENN DIAGRAMS are diagrams of usually circular or any other geometrical
shapes, representing SETS and the relationships between them being shown as
overlapping of some or all of the area.
The relationship between sets can be of different types:
* Complete inclusion of one set into another i.e. one set is a subset of another.
* Superimposition i.e. the elements of both the sets are identical.
* Partial Inclusion i.e. “some” of the elements of one set are included in
another.
* Partial Exclusion i.e.“some” of the elements of one set are excluded from
another.
* Complete Exclusion i.e. NONE of the elements of both the sets are identical.
Above relationships have been discussed in detail in this section.
A SET is a collection of distinct entities, individually specified or satisfying
specified conditions, forming a unit.
Following are various ways in which two or more sets may be related to each
other:
1. If every unit of one set is also a unit of the other set. For instance, a set of
mangoes (Set 1) is a part of a set of fruits (Set 2) or to put it in simple words
“All mangoes are fruits”
Here, “Set 1” is completely (Since the word ALL is used at the beginning of
thesentence) included or contained in “Set 2”. Besides this, there may/ may not be
other elements in Set 2.
ENGLISH USAGE

Fruits

Mangoes

In this example, if there are more elements, besides mangoes in the set of fruits,
then the shaded region will have at least one more element, other than the set of
mangoes. In other words, ‘Set of Mangoes’ will be a subset of ‘Set of Fruits’.
In case the set of fruits includes ONLY mangoes, then the shaded region will have
null value. In other words, fruits and Mangoes will be the same set i.e. the
statements…
“All fruits are mangoes” and “All mangoes are fruits”
…will have the same meaning.
And the Venn diagram will look like:

Mangoes

Fruits

2. If some but perhaps not all elements of one set are also elements of others.
For instance, a Set of teachers and a Set of Females i.e.
“Some Teachers are Females”
Here both the sets are partially contained in each other. The word “Some” is
indefinite. Does it mean “at least one” or “at least two” or “at least hundred”? For
the sake of definiteness, it is customary to regard the word “some” as meaning “at
least one”, unless specified otherwise. Thus, the above statement says that at least
one element of the set of teachers is also an element of the set of females, and vice
versa.

74
SYLLOGISM

Females Teachers

Here, Some teachers are Females and vice versa (Some females are teachers).
It may be noted here, that there may or may not be more elements, other than
those in intersection of both the sets. Does the above statement also mean “The
teachers that are not females are males”?!
Another variation of this kind of statement is
“Some teachers are not females”
This statement again, does not refer to the complete set of teachers but only to
some element or elements of the set. It says that at least one member of the set of
teachers is excluded from the whole of the set of females.
3. If two sets have no elements in common. For instance, a set of males and a
set of females i.e.
“No female is a male” or vice versa.

Males Females

CAUTION: It is advised that students do not use their existing knowledge and
preconceived notions, while doing syllogisms. The statements given have to
be assumed to be true, even if they do not hold well in real life. And that’s
where…

75
ENGLISH USAGE

THE CONFUSION BEGINS…


There are some statements given below. If the highlighted statements are true,
evaluate whether the statements following them could be known to be true or false
or will be undetermined. (Hint: make venn diagram for the highlighted statement)
Statement 1: “All cats are dogs”
1. All dogs are cats.
2. Some cats are dogs.
3. Some dogs are cats.
4. Some cats are not dogs.
5. Some dogs are not cats.
Explanation: We very well know, in real life it is impossible to believe in such a
statement. BUT, as stated earlier, to understand the syllogisms completely, we have
to shed the baggage of our existing knowledge/ preconceived notions and believe
everything stated in the given statements to be TRUE.
The given statement “All cats are dogs” can be represented diagrammatically as:

Dogs

Cats

Now, let’s have a look at each of the following statements


1. All dogs are cats- Undetermined.
Can we definitely deduce from the given information that ALL dogs are
cats?
The answer is NO… It is very much possible that there may be more
subsets of the set of dogs. Yes, in case there is no other element than the
set of cats in the set of dogs, then this statement will hold true. But we do
no have any such information. Thus, it can’t be definitely deduced from
the highlighted statement.

76
SYLLOGISM

2. Some cats are dogs- True.


We can see in the above diagram, that at least one element of the set of cats
is an element of the set of dogs. Thus, this statement can be definitely
deduced.
3. Some dogs are cats-True.
Using the reason stated in 2 above, this statement too can be definitely
deduced.
4. Some cats are not dogs-False.
Again, by looking at the diagram only, we can see that there is no element
of the set of cats that is excluded from the set of dogs. Thus, this statement
cannot be deduced at all.
5. Some dogs are not cats-Undetermined.
This cannot be definitely deduced. What if there is no other element than
the set of cats in the set of dogs?
Statement 2: “Some teachers are females”
1. Some females are teachers
2. Some teachers who are not females are males.
3. Some females are not teachers
4. Some teachers are not females

Explanation: “Some teachers are females” can be diagrammatically represented as

Females Teachers

1. Some females are teachers-True.


As we can see in the diagram itself that at least one element of the set of
females is included in the set of teachers. Thus, it can be deduced that
some females are teachers.
77
ENGLISH USAGE

2. Some teachers who are not females are males­­Undetermined.
The point to be noted here is that, can we really assume that there can be
only females and males that can be included (wholly or partially) in the set
of teachers? If we do, are we not using our existing knowledge? In the
world of syllogisms it is very much possible that the set of teachers may
include a set of animals, a set of vegetables, a set of books… and so on.
We cannot assume anything to be included in the set of teachers unless
mentioned. Thus, this statement cannot be deduced.
3. Some females are not teachers-Undetermined.
Prima facie, this statement seems to be true. It is possible that there are no
more elements in the set of females that are outside the intersection of the
set of teachers and females. In that case, it cannot be definitely deducted
from the highlighted statement.
4. Some females are not teachers-Undetermined.
Prima facie, this statement too seems to be true. Again, it is possible that
there are no more elements in the set of teachers that are outside the
intersection of the set of teachers and females. In that case, it cannot be
definitely deducted from the highlighted statement.
The basic purpose of giving syllogisms in any examination is to judge the
candidates’ capability to extract the maximum possible from the given information
ONLY. The idea is to see whether one can understand things even if he/she does
not have any prior knowledge about them.
Understanding syllogisms is helpful not only for attempting such questions,
but also to attempt any questions that require comprehension; be it Reading
Comprehension, Data Sufficiency or Data Interpretation.

SYLLOGISMS
At the first step, we should be comfortable/ conversant with finding out further
implicitly stated statements from a given SINGLE statement. As the complexity
grows, we have to use this knowledge to get the information out of MULTIPLE
statements.
Let us elaborate it with the help of examples:

78
SYLLOGISM

Example 1 : No vampires are vegetarians.


All witches are vampires.
Therefore, No witches are vegetarians.
Here, since Vegetarians are completely excluded from the set of Vampires, they
can’t be included in the set of Witches, which is a complete subset of Vampires.
The first two statements are the premises and the third statement is the conclusion
drawn from them. It can be further verified with the help of Venn diagram.

Vampires
Vegetarians
Witches

It may be noted here that the sequence of the statements is very important. The
conclusion can follow from the premise but a premise does not follow from the
conclusion.

Note:
The terms premise & conclusion have already been explained in the chapter:
Critical Reasoning.
In the following examples, we will see the difference made by different
sequencing of the statements.
Example 2 : All that glitters is gold.
My shoes are of gold.
However, here we cannot conclude from the above two statements that my
shoes glitter. They may or may not be…
Let’s see the two possible diagrammatic representation of the above statements

Glittering Things

My Shoes

Gold

It is apparent from the above diagram that the set of my shoes may or may not
be a part of the set of glittering things. Therefore, we cannot definitely conclude,
“My shoes glitter”.
79
ENGLISH USAGE

Example 3 : However, if we change the sequence of the above statements as follows:


All that glitters is gold.
My shoes glitter.
The conclusion: Therefore, My shoes are of gold, will be valid.
How? Lets see diagrammatically…
The Venn diagram would be as follows:

Glittering things

My Shoes

Gold

The change in sequence has changed the face of the diagram itself and we can
clearly see that the stated conclusion “my shoes are of gold” is absolutely valid.
Here is another example to verify that the sequence of the statements has an
impact on the validity of the conclusion.
Example 4: All rabbits are fast runners.
Some horses are fast runners.
Therefore, Some horses are rabbits.
Is that a valid conclusion?
Let’s see how the Venn diagram looks like…

Fast Runners

Rabbits

Horses

Rabbits

Fast Runners

There may be more possibilities. But, we can’t definitely conclude from the first
two statements that, “Some horses are rabbits”. They may or may not be.
Had the above Statements been
80
SYLLOGISM

Example 5 : All rabbits are fast runners.


Some horses are rabbits.
Therefore, Some horses are fast runners, would have been a valid conclusion.
The diagrammatic representation will be as follows:

Horses Rabbits

Fast Runners

Here again, it is possible to have more diagrammatic representations. But, the


conclusion in every case will come out to be the same.

TYPES OF SYLLOGISMS
Following are the type of syllogisms that are normally asked in the examinations:

TYPE 1
SIX STATEMENT SYLLOGISMS
Here each question has six statements followed by four sets of combinations of
three. Choose the set in which the statements are most logically related i.e. the third
statement follows from the first two.
Such questions require examining of each of the options one by one and
eliminate the incorrect combinations to figure out the correct answer.
Example 1 : A. All cars have four wheels
B. Santro is a car
C. Santro is a bullock cart
D. All bullock carts have four wheels
E. All cars are bullock carts
F. Santro has four wheels
a. AFB b. ADE c. ABF d. ECB

81
ENGLISH USAGE

Explanation:Let us start by examining each of the options


a. AFB
A. All cars have four wheels
F. Santro has four wheels
B. Santro is a car
The diagrammatic representation of the above statements will be as follows:

Four Wheelers

Santro

Cars

We can clearly see in the above diagram that B does not logically follow from A
& F. Also, if we think rationally, without using our existing knowledge (We very well
know that Santro is a car), we can clearly make out that just because “All cars have
four wheels” and “Santro has four wheels”; it does not mean that Santro WILL be
a car- it may or may not be. There can be something besides a car that has four
wheels and that can be Santro (which is another subset of Four Wheelers,
& not necessarily a car). Thus, this option is not the correct answer.
b.ADE
A. All cars have four wheels.
D. All bullock carts have four wheels
E. All cars are bullock carts
The same logic used for the option a. above, will also hold good here. Thus,
this option also, is not the correct answer. Students are advised to make the diagram
for this option.
c. ABF
A. All cars have four wheels
B. Santro is a car
F. Santro has four wheels

82
SYLLOGISM

Here, we can clearly see that set of cars is completely included in the set of four
wheelers and set of Santro is in turn completely included in the set of cars. Thus, we
can definitely conclude that Santro has four wheels. This can be verified by making
a Venn diagram for the same.

Four Wheelers

Cars

Santro

Although we know that there is only one correct option and we have already
figured it out, lets have a look at the fourth option to find out how it is incorrect?
d. ECB
E. All cars are bullock carts
C. Santro is a bullock cart
B. Santro is a car
This option is very much similar to option “a”. We can clearly find out the flaw
in this statement by making a Venn diagram (make it yourself!!).

TYPE 2
ROWS OF SENTENCES WITH THREE STATEMENTS IN EACH ROW
Here each question has a set of four statements. Each set has three segments.
The students are required to choose the alternative where the third segment can be
logically deduced from the first two.
Example 1: A. All men are fools; Bill is intelligent; Bill is a woman.
B. All married men are fathers; Sanjai is a father; Sanjai is married.
C. Only boys are naughty; all young are naughty; some naughty
boys are young.
D. No NRI is non-Indian; All NRIs can vote; Some Indians can vote.
a. Only A b. B and D c. Only C d. C and D

83
ENGLISH USAGE

Explanation:
CAUTION: Look at the answer choices carefully. As we can see that the
options given suggest that more than one of the given statements can be
correct. Therefore, it is required to read carefully all the statements even if
you can easily figure out that the first one is correct.
Let us start by examining the given Statements, one by one.
A. All men are fools; Bill is intelligent; Bill is a woman.
We can clearly see here that the first two segments may or may not be related to
each other. We do not know whether all fools are intelligent or all intelligent are
fools…anything is possible. But we are not aware. Thus, A is not the correct answer.
It may be noted here that such statements might seem to be true if we stop
thinking rationally. If we assume that no fools are intelligent… and if men are fools…
women would be intelligent and anyone who is intelligent is a woman!!...then we
would end up making fatal errors. We need assume nothing else, but whatever is
given in the statement to be true.
B. All married men are fathers; Sanjai is a Father; Sanjai is married.
This is an easy one!!! All married men are fathers but we can’t deduce that all
fathers are married men. Thus, even if Sanjai is a father, he may or may not be
married. This can also be verified with the following Venn diagram.

Fathers

Married men

Sanjai

Thus, Option B is not the correct answer.


C. Only boys are naughty; all young are naughty; some naughty boys are young.
The segment: ‘Only boys are naughty’ means that if there is someone who is
naughty, it has to be a boy (an element of the set of boys). There can be no one else
besides this set that is naughty. (Can we rephrase this segment, as “All naughty are
boys”?)

84
SYLLOGISM

Now, second segment states that all young are naughty. This also implies that
they are boys as well. This further implies that all young are boys and thus we can
definitely say that some naughty boys are young.
Let’s verify again with a Venn diagram.

Boys

Naughty

Young

Even though we can clearly see here that Statement C is correct, we still need to
check if statement D is correct or not as we have an answer option of C and D both.

D. No NRI is non-Indian; All NRIs can vote; Some Indians can vote.
Here segment one ‘No NRI is non Indian’, can be rephrased as All NRIs are
Indians and as per segment 2 all NRIs can vote. Thus, we can definitely conclude
that some Indians (those who are NRIs) can vote. You can verify it with Venn
diagram as well.

Indians
Voters
Voters
NRI

Indians
NRI

Thus, after looking at all the statements we can see that the correct answer
choice is “Option C and D.”
In the above question, the answer choices were such that we needed to look at
all the statements. It is always advisable to look at the answer choices first, so that
we can ascertain which statement(s) can be ignored right away.
85
ENGLISH USAGE

TYPE 3
CONDITIONAL SYLLOGISMS/ LOGICAL CONSISTENCY
In logical consistency questions students are given a main statement followed
by four statements and are required to pick a combination of two of these four
statements, which are logically correct and are consistent with the main statement
given.
There are some standard types of questions in this category.
1. If – Then type
If he is hungry, he eats pizza.
(a) He is hungry… He is eating a pizza.
(b) He ate a pizza … He was hungry.
(c) He did not eat pizza …He was not hungry.
(d) He was not hungry… He did not eat pizza.
Here statement (a) is correct (obvious)
Statement (c) is also correct, as it is the negation from the given statement.
“Had he been hungry, he would’ve eaten pizza, but since, he didn’t eat, he can
not be hungry.”
Statement (b) and (d) are not correct, as he can eat pizza without feeling hungry
too! (His girlfriend treating him and he can’t say NO!)
2. Only if- then type
He will have a pizza, only if he is hungry.
(a) He is hungry… He is eating a pizza.
(b) He ate a pizza … He was hungry.
(c) He did not eat pizza…He was not hungry.
(d) He was not hungry… He did not eat pizza.
Statement (a) is wrong, consider the case, when he doesn’t like eating pizza!
But, if the hunger is very strong and only pizza is available, does he have a
choice! In case, he had other choices available, would he have gone and eaten
pizza?
And by the above argument, statement (b) is correct.
Statement (c) is wrong, he could have been eating “vada pav”, and statement
(d) is correct.
86
SYLLOGISM

(If he dislikes pizza so much, can you force him to eat it without hunger?
Until it is a choice between girlfriend and Pizza!)
3. Either- or type- Easy ones!!!
He either Sleeps or Eats.
A. He ate.
B. He slept.
C. He did not sleep.
D. He did not eat
a. AD b. BC c. CD d. AC
Explanation: Let us have a look at the statement first. It says “ He either Sleeps or
eats”. It implies that he would perform only one of the given activities at any point
of time i.e. If he eats then he would not sleep and vice versa. Here again we start by
examining each of the options.
a. AD
A. He ate.
D. He did not eat.
We can clearly make out that the above two statements are inconsistent. It is
not possible to eat and not to eat at the same time. Thus, it is not the correct
option.
b. BC
B. He slept.
C. He did not sleep
Again, these two are also inconsistent. Thus, this is not the correct answer as
well.
c. CD
C. He did not sleep.
D. He did not eat.
Here, he seems to be doing none of the activities. Whereas, as per the statement,
in case he is not sleeping, he should have been eating and vice versa. Thus,
this too is incorrect answer.

87
ENGLISH USAGE

d. AC
A. He ate.
C. He did not sleep.
Here, since he ate, it implies that he did not sleep. Thus, this is the correct
option since both the statements are logically correct and consistent with the
main statement as well.
4. Whenever-type
This is similar to the ‘If-Then’ type.
Whenever he is hungry, he eats pizza.
This has the same connotation of
“If he is hungry, he eats pizza.”

88
SYLLOGISM

EXERCISE – 11
DIRECTION : Each question below has six statements followed by four set of
combination of three. Choose the set where the third statement can be logically
derived from first two.
1. A. All criminal actions are wicked deeds.
B. All murderers are wicked.
C. All prosecutions for murder are criminal actions.
D. All prosecutions for murder are wicked deeds.
E. All wicked people will be prosecuted.
F. All wicked people are criminals
(1) CBF (2) ACD (3) AEF (4) ABE
2. A. Stress & anxiety cause mouth ulcers.
B. Mouth ulcers are injurious to health.
C. Stress & anxiety are injurious to health.
D. Some stress and anxiety do not cause mouth ulcers.
E. Some stress and anxiety are not injurious to health.
F. Only stress and anxiety are injurious to health.
(1) CBA (2) BAF (3) BDE (4) BAC
3. A. Some dots are speckled.
B. All snakes are speckled.
C. All snakes are dotted.
D. All speckled are perforated.
E. Some dots are perforated.
F. Some non snakes are perforated.
(1) BAC (2) BED (3) ADE (4) EDC
4. A. Dots are not speckled.
B. Some dots are not speckled.
C. All speckled are perforated.
D. Some dots are perforated.
E. No dot is perforated.
F. All snakes are dots.
(1) BDC (2) CBD (3) CEA (4) EAC
89
ENGLISH USAGE

5. A. All tea planters stay in Darjeeling.


B. All hill stations are in Darjeeling.
C. All tea planters stay in hill station.
D. Manju stays in a hill station.
E. Manju stays in Darjeeling.
F. Manju is a tea planter.
(1) ABC (2) BAC (3) BDE (4) FDC
6. A. All facts are factors.
B. Some facts are figurines.
C. Some figurines are factors.
D. Some figurines may be figures.
E. Only facts are figures.
F. Some non figurines are factors.
(1) AEC (2) EBD (3) DAC (4) CAD
7. A. All jacuzzis are water springs.
B. Some jacuzzis are artificial things.
C. Volga is a water spring.
D. Volga is an artificial thing.
E. Some artificial things are water springs.
F. Volga is a jacuzzi.
(1) ACD (2) ABE (3) DCA (4) EDC
8. A. Every one who is sane can do logic.
B. All those who are Greek are sane.
C. None of your sons is sane.
D. All logicians are Greek.
E. Some Greeks can do logic.
F. None of your sons is fit to serve on a jury.
(1) ADE (2) BED (3) ABE (4) ACF

90
SYLLOGISM

9. A. All diplomats are tactful individuals.


B. Some government servants are diplomats.
C. All diplomats are corrupt.
D. Some government servants are tactful individuals.
E. Some people in public life are tactful individuals
F. All tactful individuals are corrupt.
(1) DEF (2) FCA (3) CDA (4) AFC
10. A. No woman is proud.
B. No proud person is a woman.
C. All counselors are woman.
D. No counselor is proud.
E. All women are counselors.
F. Some women are not proud.
(1) EFD (2) CAD (3) ADE (4) ECB
11. A. Violet flowers have a sweet smell.
B. All sweet smell are yellow.
C. Red roses smell sweet.
D. Canaries are yellow.
E. Red roses are yellow
F. Canaries are sweet smelling.
(1) ABC (2) BDF (3) CBE (4) CEB
12. A. Some like it hot.
B. Tim is evil.
C. Those who like it cold are evil.
D. Tim likes it cold.
E. Tim does not like it hot.
F. Those who like it cold do not like it hot.
(1) BEC (2) CEB (3) DFE (4) BDC

91
ENGLISH USAGE

13. A. All hills are mountains.


B. All rivers are valleys.
C. Some valleys are not mountains.
D. Some mountains are valleys.
E. Some rivers are not mountains.
F. All rivers are mountains.
(1) ADC (2) CEB (3) FBD (4) BED
14. A. Some elephants can fly.
B. Jumbo has wings.
C. Jumbos are elephants.
D. Jumbos can fly.
E. Those who cannot fly are not elephants.
F. Jumbos cannot fly.
(1) CDA (2) BDA (3) EFC (4) None of these
15. A. Babies are illogical.
B. Some innocent are illogical.
C. Illogical people are despised.
D. Babies are innocent.
E. No despised people can manage crocodiles.
F. Babies can’t manage crocodiles.
(1) ABD (2) ADB (3) BAC (4) FEB
16. A. All merchants are shopkeepers.
B. Some shopkeepers are union members.
C. Speculators are shrewd.
D. Some union members are not shrewd.
E. Shopkeepers are not shrewd.
F. No Shopkeepers are speculators.
(1) BDE (2) ADB (3) BAD (4) CEF

92
SYLLOGISM

17. A. All flies are mice.


B. No mice is a bird
C. Some flies are trees.
D. No fly is a bird.
E. Some flies are birds.
F. Some trees are mice
I. ABD II. DBA III. ACF IV. CED
(1) I only (2) IV only
(3) I and III only (4) III and IV
18. A. Some crows are black.
B. Some crows are not white.
C. All black are dull.
D. Some white are black
E. Some black are not white
F. All crows are dull
I. AFC II. FCA III. EAB IV. BDE
(1) Either I or II, and III (2) Either III or IV
(3) II only (4) None of the above
19. A. No animal is a dud.
B. Some duds are not intelligent.
C. Some animals are cruel.
D. No dud is an animal.
E. Some cruel are not animals.
F. Some cruel are not duds.
I. ACF II. AEF III. EAB IV. DCF
(1) I, III and IV (2) II only
(3) III only (4) I and IV

93
ENGLISH USAGE

20. A. Some buds are flowers.


B. Some twigs are flowers.
C. All twigs are branches.
D. Some flowers are buds.
E. Some branches are not twigs.
F. Some flowers are not twigs.
I. BCF II. AFD III. EBA IV. CFE
(1) I only (2) I and II only
(3) IV only (4) None of the above
21. A. All pins are clips.
B. Some clips are combs.
C. No combs are brackets.
D. Some combs are pins.
E. Some clips are not brackets.
F. Some pins are not combs,
I. FAB II. CEB III. ABD IV. EBC
(1) Either II or III (2) I and III
(3) I only (4) None of the above
22. A. Some buses are taxis.
B. Some buses are cars.
C. Some taxis are horses.
D. Some taxis are buses.
E. Some cars are not buses.
F. No car is a taxi.
I. CBD II. FAE III. EBA IV. DBF
(1) I and IV only (2) II only
(3) II and III only (4) None of these

94
SYLLOGISM

23. A. All rivers are streams.


B. No capes are oceans.
C. Some oceans are seas.
D. Some seas are not capes.
E. Some seas are capes.
F. Some rivers are streams.
I. DBE II. AEF III. ECB IV. CBD
(1) I and III only (2) IV only
(3) II and IV only (4) Either I or II and IV follows
24. A. Some lions are tigers.
B. No horses are lions.
C. Some tigers are horses.
D. Some tigers are not lions.
E. Some tigers are not horse.
F. Some tigers are lions
I. FBE II. CDB III. BED IV. CBD
(1) I and IV (2) either II or III
(3) either I or IV and either II or III (4) None of the above
25. A. Rachita wears red.
B. Rachna wears lilac.
C. Rachita likes red.
D. Everyone who likes red wears red.
E. Lilac is a dark colour.
F. Whatever is not dark is red.
I. BEF II. CAD III. ACF IV. DCA
(1) Either II or IV (2) Only III
(3) I, III and IV (4) Only IV

95
ENGLISH USAGE

EXERCISE – 12
DIRECTION : Please mark the option where the last statement follows from the
previous statements.
1. A. No peddlers are misers; some misers are millionaires; some peddlers are
not millionaires.
B. Some politicians are scoundrels; some scoundrels are absconding; some
politi cians are absconding.
C. All monks meditate; those who meditate never fall sick; all those who are
not sick are monks.
D. Girls play chess; only intelligent people play chess; girls are intelligent.
(1) Only A (2) Only D (3) A and D (4) A or C
2. A. Some boys play cricket; some boys play football; cricket is football.
B. All intelligent people pass JEE; Ramesh passed JEE; Ramesh is intelligent.
C. She travels by bus; she is rich; some rich travel by bus.
D. Some tables are not chairs; no chair is a dog; some tables are not dogs.
(1) Only C (2) Only B (3) Only D (4) A or B
3. A. EXL is a BPO; All BPOs have Reliance phones; Reliance is a vendor of
EXL.
B. Wipro is a good company; all good companies are profit centric; Wipro is
not employee centric.
C. Sea shells are expensive; all expensive things are available in Kenya; you
must get seashells in Kenya.
D. Chocolate is bad for your teeth; sweet things cause tooth decay; chocolate
is sweet.
(1) Only C (2) Only A (3) B or D (4) None of these
4. A. Water quenches thirst; thirsty Arun had a Pepsi; Pepsi is water.
B. Some musicians are pianists; some pianists are not German; some German
may not be musicians.
C. Lily hates Sillies; Sillies are good girls; Lily hates good girls.
D. Some airline customers are dissatisfied; some dissatisfied customers sued
the airline; dissatisfied customers sue their vendors.
(1) C and D (2) B and C (3) A and B (4) Only B

96
SYLLOGISM

5. A. Power tends to corrupt; knowledge is power; knowledge tends to corrupt.


B. No astrologers are scientists; some scientists are not magicians; some
magicians are not astrologers.
C. All dogs are mammals; all cats are dogs; all cats are mammals
D. No poets are painters; some artists are poets; some artists are not painters.
(1) A, C and D (2) B or D (3) Only C (4) C and D
6. A. All females are housewives; Sita is working; Sita is not a housewife.
B. Some actors can dance; all artists are dancers; no actor is an artist.
C. No tragic actors are idiots; some comedians are not idiots; some comedians
are tragic actors.
D. Diamonds are not precious stones; some carbon compounds are diamonds;
some carbon compounds are not precious stones.
(1) C and D (2) B and C (3) Only A (4) Only D
7. A. Those who eat a lot are hungry; Shivani eats a lot; Shivani is always
hungry.
B. No wealthy people are vagrants; all lawyers are wealthy; all lawyers are
vagrant.
C. All men are misogynists; Radha is a misogynist; Radha is a man.
D. All saints are immortal; Some human beings are mortal; human beings
should be immortal to become saints.
(1) A and C (2) Only B (3) B and D (4) A and D
8. A. Roses are fragrant; Jasmine is fragrant; Jasmine is Rose.
B. All Americans hate Osama; Osama is a terrorist; all Americans hate terrorists.
C. All students study; all those who study are logical; all students are logical.
D. All puddings are nice; no nice things are wholesome; puddings are not
wholesome.
(1) A and B (2) B, C and D (3) Only D (4) None of these
9. A. Some slim people do not exercise; all my sons are slim; my sons don’t
exercise.
B. Countrywide is no. 1 newspaper in Afghanistan; well-educated people
read Countrywide. Afgan are well educated.
C. Experienced people are competent; Jenkins is not competent; Jenkins is
not experienced.
D. Fido is cool; Fido drinks 7’Up; all cool people drink 7’Up.
(1) A and C (2) Only A (3) Only C (4) None of these
97
ENGLISH USAGE

10. A. No mathematicians are dumb; some dumb people are physicians; some
math ematicians are not physicians.
B. She is shrewd; she is a spin-doctor; all spin-doctors are shrewd.
C. Indians are literate; some literates lack common sense; Indians lack common
sense.
D. Kids go to school; Sita goes to school; Sita is a kid.
(1) Only A (2) Only D (3) A and C (4) None of these
11. A. Only cats meow; all dogs bark; cats are not dogs.
B. Monkeys like bananas; Tanya likes bananas; Tanya is a monkey.
C. Mobiles cause brain tumors; ultra violet rays cause brain tumors; mobiles
emit ultra violet rays.
D. All teachers teach; Brinda teaches; Brinda is a teacher.
(1) A and B (2) A or D (3) Only C (4) None of these
12. A. All ladies sing; Shipra sings; Shipra is a lady.
B. Some mangoes are sour; all grapes are sour; some grapes are mangoes.
C. All men lie; only liars are welcome; all men are welcome.
D. All fried things are heavy; some fried things are wholesome; some
wholesome things are heavy.
(1) B and D (2) A and C (3) A and B (4) Only D
13. A. Fishes stink; mushrooms stink; fishes are mushrooms.
B. All men are generous; Pranav is not generous; Pranav is not a man.
C. Lawyers wear black; doctors wear white; lawyers are not doctors.
D. Stars and sky make universe; life is a part of universe; life, stars and sky
belong to each other.
(1) Only A (2) B or D (3) Only C (4) None of these
14. A. Group of owls is parliament; politicians work in a parliament; politicians are
owls.
B. No heroes are cowards; some soldiers are cowards; some soldiers are not
heroes.
C. Animals are pets; no unicorn is a pet; unicorn is not an animal.
D. Some parrots are not pests; all parents are pets; no pets are pests.
(1) Only A (2) Only B (3) C and D (4) None of these

98
SYLLOGISM

15. A. Some poachers kill tigers; there are no tigers in Sariska; poachers live in
Sariska.
B. Sweets are fattening; all non-fried food is sweet; all non-fried food is
fattening.
C. Betting is illegal in India; Rajesh stays in Sharjah; Rajesh can do betting.
D. Good ministers are not corrupt; Mohan is not tactful; Mohan is a good
minister.
(1) Only B (2) A or C (3) Only D (4) None of these
16. A. All managers are leaders; all politicians are leaders; some politicians are
leaders.
B. Gangsters smuggle; gangsters are criminals; all those who smuggle are
criminals.
C. Metropolitans have good infrastructure; Chappra has good infrastructure;
Chappra is a metropolitan.
D. Red is blue; blue is black; red is black.
(1) Only B (2) B and D (3) A or C (4) Only D
17. A. Hot coffee is good; hot coffee is carcinogenic; some carcinogenic things
are good.
B. Ghosts do not scare me; ghosts do not scare brave people; I am brave.
C. All pillows are bed sheets; some bed sheets are not radios; some pillows
are not radios.
D. Catches win matches; Sidhu won the match; Sidhu takes good catches.
(1) A, B and C (2) B and C (3) A or C (4) Only C
18. A. Whales are not animals; No mammals are animals; Mommals are not whales.
B. Whales are mammals; No mammals are fishes; Whales are not fishes.
C. Whales and fishes live on trees; Fishes are not mammals; Whales are not
mammals.
D. Mammals live in water; Whales and fishes live in water; Whales are not
fishes,
(1) Only A (2) Only B (3) Only C (4) Only D

99
ENGLISH USAGE

19. A. Some plants are thorny; Some plants are flowery; Thorny are flowery.
B. All thorny are leafy; All leafy are flowery; Some flowers are thorny.
C. Bat is viviparous; No mammal is a bird; Bat is not a mammal.
D. Bat is viviparous; Bat is a mammal; Some mammals are viviparous.
(1) B and C only (2) A and B (3) B only (4) B and D
20. A. Some fruits are sour; Some apples are sour; All sour are either fruits or
apples.
B. Some As are B’s; All C’s are A’s; Most C’s are B’s.
C. All birds lay eggs; An owl is a bird; An owl lays eggs.
D. All A’s need B; C’s are A’s; C’s need B.
(1) A and C (2) B and D (3) C and D (4) C only
21. A. All animals need sleep; Men need sleep; Men are animals.
B. Dogs are animals; Men are animals; Some men are dogs.
C. All animals need sleep; Men are animals; Men need sleep.
D. All animals need sleep; dogs are animals; dogs need sleep.
(1) B and C (2) C and D (3) A and C (4) C only
22. A. Tomatoes are fruits; Some tomatoes are not sour; Some fruits are sour.
B. An ostrich lays eggs; An ostrich is a bird; All birds lay eggs.
C. Kiwi is a bird; Some birds can fly; Kiwi cannot fly.
D. Oranges are fruit; Some oranges are sour; Some fruits are sour.
(1) A, C and D (2) B and D (3) D only (4) A and D
23. A. Radha is nasty; All girls are nasty; Radha is a girl.
B. Marutis are Fiats; Maruti is the oldest car in India; Fiat is one of the oldest
cars in India.
C. Theory is practical, Anatomy is practical; Anatomy is theory.
D. Raju is a villager; Raju is an actor; Some villagers are actors.
(1) B and C (2) Only D (3) B and D (4) None of these

100
SYLLOGISM

24. A. Films are dramas; Dramas are nonsense; All nonsense are films.
B. Reading increases knowledge; writing improves reading; writing improves
knowledge.
C. No rats are cats; This is a cat; This is not a rat.
D. Religion is violence; Religion teaches non-violence; Some violence teaches
non-violence,
(1) A (2) B, C (3) B, C, D (4) A, C, D
25. A. All hill stations are above sea level; Mahabaleshwar is 8000ft. above sea
level; Mahabaleshwar is a hill station.
B. Rainbow has seven colours; Preity is wearing a seven-coloured dress;
Preity is wearing rainbow.
C. All teachers are ethical; Rajesh is ethical and works in a school; Rajesh is
a teacher.
D. No Martian is fair; Jaadoo is not a Martian; Jaadoo is fair.
(1) C and D (2) A or C (3) B or D (4) None of these

101
ENGLISH USAGE

EXERCISE – 13
DIRECTION : Given the 1st statement of the option is true, mark which pair logically
follows from the given statement.
1. Ronnie drinks either tea or juice.
A. Ronnie does not drink tea.
B. Ronnie drinks tea.
C. Ronnie drinks juice.
D. Ronnie does not drink juice.
(1) AB (2) CD (3) AC (4) BC
2. He drinks rum and milk together.
A. He did not drink rum.
B. He drank rum.
C. He did not drink milk.
D. He drank mango milk.
(1) AC (2) AB (3) AD (4) CD
3. He neither Jumps nor Runs.
A. He jumped.
B. He did not run.
C. He did not jump.
D. He ran.
(1) AB (2) BC (3) CD (4) DA
4. Rose is either a thorn or flower.
A. Rose is a thorn.
B. Rose is a flower.
C. Rose is a butterfly.
D. Rose is not a flower.
(1) CD (2) DA (3) BC (4) AB
5. Either Paresh or Ashu is intelligent.
A. Paresh is intelligent.
B. Ashu is intelligent.
C. Paresh is not intelligent.
D. Ashu is not intelligent.
(1) AC (2) BD (3) BA (4) AD
102
5
Questions Related to Words
Word related questions could be mark fetching, less time consuming and easier
to handle if your command over vocabulary is good. Such questions check your
knowledge of words and their applications. So, it is essential that you should
continue to enhance you word stock. Reading regularly is the best method for the
improvement of your vocabulary. You can also take help of the Wordlist, which is
provided to you with the study materials.
Along with this, you also need to develop your logical sense. You would be
asked to identify the connotation of the words in relation to their likeness, contrast
etc. with other words.
There are two kinds of word related questions which we will be dealing below.
1. Odd Man Out
2. Analogy
3. Words with multiple usage / meanings

ODD MAN OUT


As the name itself suggests, in such type of questions, you are required to pick
out the odd word, the word which is anomalous among other words in the options.
The other words have an inherent relationship with one another, that is, they could
be labeled in a class that the odd word could not. It is thus imperative to…
1. know the meaning of the words
2. identify the basis on which the words fall within a class.
3. pick out the odd word which defies that class.
Let us understand it better with some examples.
Example 1 :
1. ontology 2. anthology 3. philology 4. geology
ENGLISH USAGE

Solution : The question could prove confusing if you do not know the meaning of
the words. All the words are ending with ‘-logy’. We know that generally the words
having suffix ‘-logy’ connote ‘science’. So, if you don’t know the meanings of the
words here, you would presume that all the words are names of sciences. But it is
true of 1], 3] and 4]. The word ‘anthology’ means a collection of poems, essays etc.
So, the answer is 2]
Example 2 :
1. carrot 2. parsnip 3. cucumber 4. radish
Solution : We know that all the words are the names of agricultural products. Now,
stop here and before reading any further try to think which is dissimilar from the
other words and why. Got it? Yes, we can see here that ‘carrot’, ‘parsnip’ and
‘radish’ grow underground whereas ‘cucumber’ grows on the ground. So, the
answer is 3]
Example 3 :
1. sinusitis 2. influenza 3. typhoid 4. malaria
Solution : Here again, we find that all words are the names of diseases. But ‘influenza,
typhoid and ‘malaria’ are bacterial or viral diseases and they cause fever. But
‘sinusitis’ does not cause fever and it is not bacterial or viral too. So, the answer
is 1]
Example 4 :
1. slumber 2. nap 3. snooze 4. saunter
Solution : All the words here except ‘saunter’ means ‘ to sleep’. ‘Saunter’ means to
walk leisurely. So, the answer is 4]
Example 5 :
1. petrol 2. coal 3. wood 4. fire
Solution : Words in options 1], 2] and 3] are fuels. They burn and give out fire. As
‘fire’ in option 4] is the product, it does not belong to the same class. So, the answer
is 4]

104
QUESTIONS RELATED TO WORDS

EXERCISES – 14
1. (1) Veracity (2) Mendacity (3) Perjury (4) Fabrication
2. (1) Rogue (2) Vegabound (3) Hooligan (4) Mendicant
3. (1) Drab (2) Humdrum (3) Prosaic (4) Arduous
4. (1) Tangential (2) Basic (3) Superficial (4) Cursory
5. (1) Gerrymander (2) Obfuscate (3) Misconsture (4) Expedition
6. (1) Beat (2) Kill (3) Goad (4) Mane
7. (1) Aristotle (2) Einstein (3) Plato (4) Socrates
8. (1) Spiritual (2) Devout (3) Pious (4) Atheist
9. (1) Extrovert (2) Reticent (3) Amiable (4) Gregarious
10. (1) Hoof (2) Whine (3) Equine (4) Gaunt

ANALOGY
Analogies test your ability to identify the relationships of words. You are given
a pair of words. The words are related to each other in a particular way. The options
have four pair of words. The pairs are also related to each other in some way. Your
job is to identify the pair of words from the options which are related to each other
in the same manner as the pair in the question. The process to handle such questions
is to
1. identify the relationship between the pair of words in the question.
2. identify the relationships between the pairs of words in the options.
3. pick out the pair from the option which is related in similar manner as the
pair in the question.
Some of the relationships could be
1. Cause and Effect 2. Part and Whole
Accident : Death Nail : Finger
3. Synonym 4. Antonym
Marvelous : Wonderful Cheerful : Gloomy
5. Thing and Action 6. Professional and Work
Pen : Write Lecturer : Teach
7. Class and Member 8. Object and Characteristics
Wardrobe : Shirt Nightingale : Bird Dacoit : Cruel
105
ENGLISH USAGE

9. Qualitative Relationship 10. Worker and Workplace


Translucent : Opaque Blacksmith : Forge

Example 1 :
DUNCE : GENIUS
1. Man : Mammal
2. Waste : Conserve
3. Truthful : Artful
4. River : Sea
Solution : Can you identify what is the relation between the words in question? Yes,
they have an antonymic relation. A‘ dunce’ is a ‘stupid person’ whereas a ‘genius’
is a ‘highly intelligent person’. Now see, which of the pair in the options has
antonymic relationship. We see that option 2] has such relation. So, the answer
is 2]
Example 2 :
CRUST : PIZZA
1. Food : Restaurant
2. Cold drink : Burger
3. City : Village
4. Car : Engine
Solution : It is not hard to see that crust is a part of pizza. Thus, the relation in which
the sample pair stand is that of Part and Whole. What pair among the options do
you find having the same relation? Don’t get confused by the options 1] and 2]
because they also contain food items. Their relation is not that of the sample pair.
Rather, we find option 4] having the same relation. So, the answer is 4]
Example 3 :
BRIGHT : SUN
1. Sharp : Sword
2. Light : Moon
3. Stars : Night
4. Honey : Bee

106
QUESTIONS RELATED TO WORDS

Solution : Brightness is the characteristic of the sun. We have known the relation
then. Now, among the options we find that the pair in option 1] share the same
relation. A sword is sharp. So, the answer is 1]
Example 4 :
EXHIBIT : DISPLAY
1. Show : Show off
2. Hare : Rabbit
3. Feel : Good
4. Know : Understand
Solution :One can easily see that the sample pair stands in synonymous relation.
Both ‘exhibit’ and ‘display’ means ‘to show’. In option 1] ‘show’ and ‘show off’ do
not have the same meaning. ‘show off’ has a negative connotation but ‘show’ does
not. We find the synonymous relation between the words in option 2]. Both ‘hare’
and ‘rabbit’ have the same meaning. So, the answer is 2]
Example 5 :
CAR : IRON
1. Computer : Screen
2. Steel : Iron
3. Table : Wood
4. Bus : Road
Solution : What relation do you find here? A ‘car’ is made of ‘iron’. We find ‘table:
wood’ having the same relation. One can find somewhat similar relation in option 2]
as well. But you should notice that ‘steel’ is made from ‘iron’ and not of ‘iron’. Thus
there is a difference. So, the answer is 3]

107
ENGLISH USAGE

EXERCISES – 15

1. FOLLOW : STALK 6. DECREPIT : RENOVATION


(1) Regret : Rejoice (1) Enervated : Invigoration
(2) Look : Spy (2) Languid : Confrontation
(3) Lurk : Hide (3) Pallid : Purification
(4) Beckon : Gesture (4) Tearful : Reconciliation
2. DESCRY : DISTANT 7. GRISLY : RECOIL
(1) Mourn : Lost (1) Craven : Quail
(2) Whisper : Muted (2) Tainted : Purify
(3) Discern : Subtle (3) Flagging : Invigorate
(4) Entrap : Hostile (4) Heartrending : Weep
3. RUSE : DECEIVE 8. LUMINARY : ILLUSTRIOUS
(1) Policy : Change (1) Zealot : Intense
(2) Argument : Persuade (2) Miser : Prodigal
(3) Subterfuge : Revenge (3) Dignitary : Conceited
(4) Denial : Confuse (4) Celebrity : Wealthy
4. CENSURE : REPREHENSIBLE 9. BEAM : DELIGHT
(1) Prize : Valuable (1) Frown : Insidious
(2) Provide : Supportive (2) Glower : Anger
(3) Applaud : Enthusiastic (3) Yawn : Assurance
(4) Inquire : Informed (4) Grin : Compassion
5. INADVERTENT : THOUGHT 10. SCOTCH : RUMOUR
(1) Gauche : Grace (1) Divert : Traffic
(2) Lugubrious : Gloom (2) Broach : Topic
(3) Curious : Opinion (3) Suppress : Riot
(4) Clandestine : Secrecy (4) Spread : Gossip

108
QUESTIONS RELATED TO WORDS

WORDS WITH MULTIPLE USAGE / MEANINGS


In English certain words can be used in multiple ways. In the recent years many
questions related to multiple usage of words have been asked in the CAT exam.

EXAMPLES
1. BLOW

T h e d ev o te es p r aye d fo r th e sc an d al o v e r th e K an ch i m u tt to
1.
b lo w o v er so o n .
S u spectin g him to b e d ru n k, th e po licem an asked th e d river to
2.
b lo w the breath analyzer.
3. T he car sp u n o ut o f co n trol an d crash ed the sid e o f a tru ck.

4. H e r fa th e r’s d e a th ca m e as a ter rib le b lo w to R e va th i.

Answer (2)
Except for (2), all other usage of BLOW are correct. Sentence (2) should read:
“... the policeman asked the driver to blow into the breath analyser.”
2. MAINTAIN
He continued to maintain that he
Keep something in good
A E was innocent even after compelling
condition
evidence had been presented.
To make something continue at His new job provided him barely
B F
the same level enough to maintain a family of four.
The government has struggled to
Keep stating that something is
C G maintain the prices of essential
true even when others disagree
commodities this year.
Support somebody financially While its initial cost is low, this car
D H
over an extended period takes a lot of money to maintain.

(1) (2) (3) (4)


A H A G A H A F
B E B H B G B G
C G C F C E C E
D F D E D F D H
Answer (3)

109
ENGLISH USAGE

EXERCISE – 16
DIRECTIONS: In each question, the word at the top of the table, is used in
four different ways, numbered 1 to 4. Choose the option in which the usage of
the word is INCORRECT or INAPPROPRIATE.
1. CRASH

1. The fall of the government caused a share market crash.


2. A crash course in computer programming was conducted for the employees.

3. The car spun out of control and crashed the side of a truck.

4. Vijay crashed to a 0-6 defeat in the final set of the tennis match.

2. SAMPLE

1. The aquarium has some interesting samples of rare tropical fishes.

2. The survey was based on a random sample of middle-class households.

3. The company gave a free sample of shampoo with every purchase of its soap.

4. I sampled the dishes prepared for the party.

3. LUXURY

1. Rohit has landed a plum job, now he will be able to live in the lap of luxury.
2. A washing machine was considered luxury half a century ago.
3. I allow myself the luxury of visiting my parents every week.
4. A long warm bath is one of life's little luxuries.

4. EXPLODE
1. Bombs seem to explode at regular intervals in some parts of India.
2. This report explodes the myth that thin is beautiful.
3. After ten minutes, the game exploded with life.
4. He explodes into a torrent of abuse at the slightest provocation.

110
QUESTIONS RELATED TO WORDS

5. TRICK

1. Even his entreaties, which looked genuine, failed to do the trick for him.

2. The boy was up to his tricks again, after the teacher left the room.

3. Everyone knows that offering a discount is a trick of the trade.


The unsuspecting villager was tricked to believing that the stranger was a
4.
policeman.

6. INCH

1. Australia was within an inch of victory when rain stopped play.


2. The army inched forward towards the enemy country's capital.
3. The party was increasing its tally of seats inch by inch.
4. He looked each and every inch a wealthy aristocrat.

7. PUSH

1. The police tried to pu sh their way into the flat.

2. N o one could push her to join the army.


In spite of the unfavourable weath er they were determined to
3.
push towards the East coast.
4. The students may p ush for a postponem ent o f their exam s.

8. WASH
1. Y ou r film sy excuses w ill no lo n ger w ash w ith m e.
2. D on ’t w ash in yo ur fo o d w ith w ater; it is n o t a g oo d h ab it
3. T h e sem i-fin als o f th e B & H C u p w ere w ash ed ou t.
4. W h y d o n’t yo u h ave a q u ick w ash w h ile I serve d in ner?

DIRECTIONS for questions 9 to 20 : For the word given on the top of the
table, match the dictionary definitions given in the left hand columns (A, B, C,
D) with their corresponding usage given in the right hand column (E, F, G, H).
Out of the four numbered choices given in the boxes, identify the one that has all
definitions and usages correctly matched.
111
ENGLISH USAGE

9. STRIKE

A period of time when a The funny side of the argument


A E
group of workers stop work suddenly struck me.

A military attack, especially During the lecturers' strike, all


B F
by aircraft dropping bombs colleges were closed.
To hit or knock hard against American aircraft launched
C G
something several strikes on Afghanistan.
To suddenly realise that
He was digging when his shovel
D something is important or H
struck against a metallic object.
interesting

(1) (2) (3) (4)


A F A F A G A G
B E B G B H B E
C G C H C E C F
D H D E D F D H

10. SHOOT

To direct something The police have tried in vain to


A E
suddenly or hastily shoot the fugitive criminal.
The children were ready to shoot
B Moving or growing quickly F
questions at us.
To be very critical of someone’s I have a shooting pain in the
C G
opinion back.
The scientist’s latest theories have
D To kill or wound someone H
been shot down by his critics.

(1) (2) (3) (4)


A G A F A F A G
B F B H B G B H
C E C G C H C E
D H D E D E D F

112
QUESTIONS RELATED TO WORDS

11. CHAPTER

We hope that his departure from the


A Separate section of a book E
city will close this unhappy chapter.

All the priests of a church or The chapter will meet soon to discuss
B F
a religious community issues concerning the church.

The Chandigarh chapter of the IMA


C An unpleasant episode G
met today.
Though I did not enjoy the book
Local branch of a club or
D H much, the chapter on animal
society
intelligence caught my attention.

(1) (2) (3) (4)


A H A H A G A E
B F B F B F B F
C G C E C E C G
D E D G D H D H

12. ROUGH

The ship had to endure rough


A Coarse, indelicate E
weather.
We had to rough it out in the army
B Inexact, approximate F
bunkers.
Can you give me a rough idea as to
C Stormy, tempestuous G
how much time it will take?

To live without
His rough language does not endear
D customary comforts and H
him to our friends.
conveniences.

(1) (2) (3) (4)


A F A G A H A H
B G B H B G B G
C E C E C F C E
D H D F D E D F

113
ENGLISH USAGE

13. SUPERFICIAL

The Board members were engaged in


Not studying something
A E superficial chatter before the meeting
thoroughly
commenced.
A superficial reading is just not enough
B Appearing to be true F
to understand this article.
Often, we see people giving a superficial
C Not serious G
impression of warmth and friendliness.
She came back to normalcy soon as the
D On the surface, not deep H injuries sustained in the accident
were only superficial.

(1) (2) (3) (4)


A G A F A F A G
B F B E B G B F
C E C G C E C H
D H D H D H D E

14. LIBERAL
Willing to understand and I am n ot su rprised that V ijay is so
A respect the behaviour and unpopular as he is very liberal with his
opinions of others E sarcasm.
The 1960s were characterised by the
Generous, given in large"
B F prevalence of liberal views on the use of
amounts
psychedelic drugs.
Abraham Lincoln was a liberal who felt
Not completely accurate or
C G t h a t s l a v e r y b e s m i r c h e d A m e r i c a 's
exact
image.

This liberal interpretation of the causes


A person who advocates
and consequences of World War II, does
D political, social and H
a disservice to those who have no prior
religious freedom
knowledge of this cataclysm.

(1) (2) (3) (4)


A F A F A E A F
B E B G B F B E
C H C H C H C G
D G D E D G D H

114
QUESTIONS RELATED TO WORDS

15. STABLE

Firmly fixed, not likely I think all of them are actors from
A E
to move or change the same stable.
Though he is quite experienced, he
B Staying in the same state F
does not seem to be mentally stable.
Prices of electronic goods have
C Calm and reasonable G
remained stable for a few years now.
Persons, products having You have to prove that the product
D H
a common origin obtained is a stable compound.

(1) (2) (3) (4)


A H A G A H A G
B F B F B G B H
C E C H C F C F
D G D E D E D E

16. NARROW

His analysis was different from mine


Measuring a short distance
A E because he considered the word
from one side to the other
"culture" in a narrower sense than I did.
We were forced to drive very slowly, as
B With little margin F
the streets were very narrow.
Limited in a way that
She had a narrow escape when her car
C ignores important issues or G
skidded.
the opinions of other people
Her attitude reflects her narrow view of
D Limited in meaning; restricted H
the world.

(1) (2) (3) (4)


A G A G A F A F
B H B F B G B G
C E C H C E C H
D F D E D H D E

115
ENGLISH USAGE

17. RATE

Estimate the value or The question was so silly, that it did


A E
worth of not rate a serious reply.
B Merit F This advertisement is first-rate.
We rate the Australian cricket team
C Class or rank G
highly.
Rapidity of movement At this rate, Koneru Humpy will be a
D H
or change world-beater in chess.

(1) (2) (3) (4)


A H A G A G A G
B E B F B E B E
C F C E C H C F
D G D H D F D H

18. GENERATION

All the people born


The W adias have ruled the world of
A during a particular E
business for three generations.
period of time
He has taken to illegal methods of
A single step in descent
B F income generation to make both
or pedigree
ends meet.
Though the present generation is
Production by natural
C G technically skilled, it has lost its
or artificial process
cultural m ooring
A group of people of
She has inspired a whole generation
D sim ilar age involved in H
of techies with her innovative ideas.
a particular activity

(1) (2) (3) (4)


A G A G A E A E
B E B E B G B H
C F C H C H C F
D H D F D F D G

116
QUESTIONS RELATED TO WORDS

19. INVENTION

The ability to develop The internet has proved to be a


A E
new and interesting ideas remarkable invention.
The act of inventing The invention of the printing press
B F
something catalysed the Renaissance.
The precocious child was full of
C A story that is untrue G
invention.
A thing or idea that has His explanation for his absence was a
D H
been created complete invention.

(1) (2) (3) (4)


A G A H A G A F
B F B E B F B G
C H C G C E C E
D E D F D H D H

20. REGISTER

To show on a measuring I formally register my disapproval of


A E
instrument the measures suggested.
The level and style of a piece of I suspect the note was played in the
B F
writing or speech middle register of the cello.
Part of the range of a musical The earthquake registered 1.8 on the
C G
instrument Richter scale.
To make an opinion known The new book surprised me with its
D H
publicly rather informal register.

(1) (2) (3) (4)


A G A G A H A F
B H B F B G B G
C F C H C R C R
D E D E D F D H

117
6
Common Misspellings

Some words are easy to spell. Some aren’t. Some are spelled like they sound.
Some aren’t. The following is a list of frequently misspelled words. In some cases, a
particularly common misspelling is given in parentheses. Sometimes these alternate
spellings are listed in permissive dictionaries as spelling “variants,” but this does
not mean these spellings are correct spellings. (The word ain’t is in the dictionary,
too, but it’s not correct.)
accelerate barbecue (barbeque)
accidentally (accidently) believable (believeable)
accommodate believe
accordion (accordian) broccoli
accumulate camouflage
acquaintance (acquaintence, cantaloupe
aquaintance) carburetor
acquire (aquire) Caribbean
acquit (aquit) dissipate
aficionado drunkenness (drunkeness)
a lot (alot) dumbbell (dumbell)
amateur ecstasy
anoint embarrass
apology exercise (excercise)
argument (arguement) exhilarate
atheist existence (existance)
a while (awhile) Fahrenheit
axle (axel) fiery (firey)
COMMON MISSPELLINGS

flabbergast (flabberghast) judgment (judgement)


flotation (floatation) led (lead)
frustum (frustrum) liaison
gauge lieutenant
genius (genious; see liquefy (liquify)
“ingenious”) lose (loose)
grammar (grammer)
maneuver, also manoeuvre
gross
renowned
guttural
rhythm
handkerchief (hankerchief)
ridiculous
harass (harrass)
sacrilegious
horrific
sandal
hypocrisy
savvy
imitate
seize (sieze)
immediately
sensible (sensable)
inadvertent (inadvertant)
incidentally (incidently) separate
incredible septuagenarian (septagenarian)
independent sergeant
indispensable (indispensible) sheriff
ingenious (ingenius; see shish kebab
“genius”) siege
inoculate (innoculate) similar
irascible simile
irresistible (irresistable) sorcerer (sorceror)

NEGATIVES WITHOUT POSITIVES


In English, the prefixes in and un, along with others, are used to negate certain
root words. The antonym of sane, for example, is insane, and the antonym of
stoppable, for example, is unstoppable. But some words that appear to be negated
with prefixes have no positive. The opposite of inept is not ept — there is no such
word.

119
ENGLISH USAGE

WORDS WITH NO POSITIVE FORMS


debunk inhibited
defenestrate insidious
dejected insipid
disconsolate insouciant
disgruntled intact
dishevelled invert
dismayed misgivings
feckless misnomer
gormless nonchalant
impetuous noncommittal
impromptu nondescript
inane nonpareil
incessant nonplussed
inchoate unbeknownst
incognito ungainly
incommunicado unnerved
indomitable unswerving
ineffable untold
inept untoward
inert
infernal

WORDS WITH UNCOMMON POSITIVE FORMS


The following words do have positives, formed by removing the negating
prefix, but the positive forms of these words are far less common:
disarray insufferable
disconcerting interminable
immaculate unbuiddled

120
COMMON MISSPELLINGS

impeccable unflappable
inadvertent unfure
incapacitated unkempt
incorrigible unarittigated
inevitable unrequited
innocent unruly
inscrutable unthinkable
insensate unwieldy

COMMONLY CONFUSED WORDS


ABJURE AND ADJURE
Abjure means to “renounce” or “repudiate.” Adjure means “request earnestly.”
The words have little in common other than their rarity and similarity of spelling, so
they should not be confused.

ADOPTED AND ADOPTIVE


Adopted and adoptive both mean “acquired through adoption,” but they are
not synomyms. One is the reciprocal of the other. A child with adoptive parents is
adopted; parents with an adopted child are adoptive. The difference lies in who is
doing the adopting. The parents adopt the child, so the child is adopted. The child
is adopted by the parents, and so the parents are adoptive.

ADVERSE AND AVERSE


Adverse means “antagonistic.” Averse means “feeling disinclined.” The word
averse is more recognizable in two more commonly used forms: avert and aversion.

AFFECT AND EFFECT


These words are commonly confused, and the rules given to tell them apart are
often wrong. Affect means “to influence” or “to produce an effect in.” Effect means
“consequence” or “that which is produced by an agent or cause.”

121
ENGLISH USAGE

Many try to differentiate between the two by saying that affect is a verb, while
effect is a noun. Unfortunately, it’s not so simple. The word affect can also serve as
a noun, meaning “observed or expressed emotional response,” and the word effect
can also serve as a verb, meaning “to become operative” or “to carry out,” as in, “to
effect changes.”

AGGRAVATE AND ANNOY


Aggravate means “worsen.” Annoy means “bother” or “exasperate” or
“provoke.” Many speakers and writers use aggravate to mean “annoy.” Although
aggravate has been used in this manner for four hundred years, considerable
controversy over this use exists today. Some contend that using aggravate to
mean anything other than “worsen” compromises the effectiveness of the word by
blurring the distinction it has from similar words. Others argue that annoy can be
said to mean “worsen one’s temper,” which suggests that aggravate is not so
inappropriate to use as a synonym for annoy after all. My recommendation is to
understand that aggravate means “worsen” and not “bother,” but then feel free to
use aggravate in contexts where it would be taken to mean “worsen one’s temper”
rather than “bother” or “irritate.”

ALLUDE AND REFER


Allude means to refer to something indirectly or covertly. Refer, without
qualification, implies referring to something directly, by naming. Frequently allude
is misused to mean “refer directly,” but this is an abuse of an otherwise useful,
specific word.

ANXIOUS AND EAGER


Anxious means “troubled” or “worried.” Eager means “having keen interest”
or “impatient expectancy.” When anxious is used to describe someone’s expectancy,
it is often incorrectly used to mean “eager,” which has a far more positive connotation.
One may be anxious about an impending report card, but one would be eager to go
on a long-awaited vacation.

122
COMMON MISSPELLINGS

CHAFE AND CHAFF


Chafe means “to make sore by rubbing” or “irritate or annoy” or “become
annoyed.” Chaff means “tease good-naturedly.” Consequently, one should not be
confused with the other.

COMPLIMENT AND COMPLEMENT


Compliment is a “remark of praise.” Complement is “something that completes.”
Note that free refreshments are complimentary; the word refers to the phrase “with
our compliments.”

CONTINUAL AND CONTINUOUS


Both continual and continuous describe an action or process that occurs over
a long period of time. Continual, however, permits that the action may be interrupted
by short breaks. Continuous means that the action never pauses. We live continuous
lives, eating and sleeping continually.

DESERTS AND DESSERTS


Most English speakers understand that deserts, with the accent on the first
syllable, are a dry, arid lands, while desserts, with the accent on the second syllable,
are a sweet things to eat after a meal. What is often confused is that when one gets
what one is deserves, good or bad, one is getting one’s “just deserts,” accent on the
second syllable but spelled like the dry, arid lands.

DISCOMFORT AND DISCOMFIT


Discomfort means “uneasiness or hardship” and “make uncomfortable.”
Discomfit means “disconcert” and “defeat; thwart.” Discomfort comes from Middle
English, from the Old French word desconfort. Discomfit comes from the Middle
English word discomfiten, from the Old French word desconfit.

DISCRETE AND DISCREET


Discrete means “discontinuous” or “individually distinct.” Discreet means
“judicious.” The words are understandably often confused, but they should remain
distinct.
123
ENGLISH USAGE

EFFECTIVE AND EFFECTUAL


Effective means “producing an effect” or “in effect.” Effectual means “producing
a desired effect.” A law that is effective is only effectual when it is enforced.

ENERGIZE AND ENERVATE


Energize means “give energy to,” while enervate means “to cause to lose
vitality or energy.” Sometimes enervate is mistakenly believed to mean “energize,”
and this is a grievous error, as enervate is actually an antonym of energize.

ENORMOUSNESS AND ENORMITY


Enormousness means “largeness” or “immensity.” Enormity means “depravity”
or “wickedness” and also means a “crime” or “error.” Frequently the word enormity
is mistaken to mean “enormousness,” which it does not. The two words do originally
derive from the same Latin word, enormis, with the word enormity being derived
through the French word enormite, but the distinction between the two English
words has existed for hundreds of years.

FLACK AND FLAK


Flack is a noun that means “press agent” or “publicist” and a verb that means
“to act as a press agent.” Flak is a noun that means “anti-aircraft artillery” or
“bursting of shells fired form anti-aircraft artillery” or, more commonly, “excessive or
abusive criticism” or “dissension, opposition.” If you criticize someone, you are
giving them flak, not flack.

FLAUNT AND FLOUT


Flaunt means “display ostentatiously.” Flout means to “disobey openly and
scornfully” or “show contempt for.” Some use one for the other, but confusing the
two words is still widely seen as an error and should be avoided.

FORCIBLE AND FORCEFUL


Forcible means “accomplished by force.” Forceful means “powerful and
vigorous.” Something that is forcible can be accomplished with forceful effort.

124
COMMON MISSPELLINGS

FORTUITOUS AND FORTUNATE


Fortuitous means “happening by chance” or “accidental.” Fortunate means
“lucky.” A fortuitous event may be, but is not necessarily, a fortunate event.

FOUNDER AND FLOUNDER


Founder means “to fail completely.” Often it is used to refer to a ship, in which
case it means, “to fill with water and sink.” Flounder has a less severe definition; it
means “to move clumsily or with difficulty” or “to make mistakes or become
confused.” Although the two words are similar in spelling, they are derived from
different sources. Founder comes from the Latin word fundus, meaning “bottom.”
Flounder comes from Middle English, from the Anglo-Norman word floundre, of
Scandinavian origin. We can flounder but recover, but when we founder, it’s final.

FULL AND FULSOME


Fulsome is sometimes mistaken as a synonym for full or fullest, when, in fact,
its meaning bears no resemblance to full whatsoever. Fulsome means “cloying,
excessive, and disgusting.” Although both words have similar etymological roots,
the English definitions are divergent enough to cause some gross misunderstandings
when the words are confused, particularly in incorrect expressions such as “fulsome
praise” and “fulsome apologies.”

IMPLY AND INFER


Imply means to “hint or suggest without stating directly.” Infer means “reach
an opinion from facts or reasoning.” The two terms are sometimes mistaken to be
interchangeable. In actuality, they are quite distinct. The sender of an indirectly
stated message is doing the implying, while a receiver that reasons what the message
is, is doing the inferring.

INANITY AND INANITION


Inanity means “foolishness” or “senselessness.” Inanition means “lacking
vigor.”

INCREDULOUS AND INCREDIBLE


Incredulous means “unbelieving” or “skeptical.” Incredible means
“unbelievable” or “hard to believe.” If something incredible happens, you may be

125
ENGLISH USAGE

incredulous Nauseated means “caused to feel nausea.” Nauseating means “causing


to feel nausea.” If you are nauseated, it is because something else is nauseating.
There is no controversy about the meaning of these words, but there is much
controversy over what nauseous means. Commonly, the word nauseous is used to
mean “nauseated,” but some maintain that it should only ever be used to mean
“nauseating.” In actuality, the word nauseous has meant both for over 400 years,
and there is little reason to dispute an accepted usage of a word that dates back to
the times in which the English language was at the summit of its development.

NOISOME AND NOISY


Noisome means “harmful” or “offensive” or “disgusting.” Obviously it does
not share any shade of meaning with noisy, and so the two words should not be
confused with one another.

PASSED AND PAST


Passed is a past tense verb. Past can be a noun, adjective, adverb, or preposition
but never a verb.

PECCABLE AND PECCANT


Peccable means “capable of sinning.” Peccant means “guilty of sinning.”

PERSPICACIOUS AND PERSPICUOUS


Perspicacious means “having or showing insight.” Perspicuous means “easily
understood” or “lucid,” or, when referring to a person, “expressing things clearly.”

PRACTICABLE AND PRACTICAL


Practicable means “usable,” while practical means “useful.” Not all practicable
things are practical, and not all practical things are practicable.

SANGUINE AND SANGUINARY


Sanguine means “hopeful” or “optimistic” or “confident.” Sanguinary means
“bloody” or “murderous.” Obviously these are two words it’s better not to confuse,
lest you convey entirely the wrong idea about something.

126
COMMON MISSPELLINGS

STANCH AND STAUNCH


Stanch is a verb that means “restrain a flow.” Staunch is an adjective that
means “firm in attitude, opinion, or loyalty.” Both words are pronounced the same
way. The distinction is actually quite recent; a hundred years ago, the two words
were treated as interchangeable spelling variants. Such is no longer the case, so it’s
important to distinguish one from the other.

STATIONARY AND STATIONERY


Stationary is an adjective that means “not moving.” Stationery is a noun that
means “writing paper and envelopes.”

TITILLATE AND TITIVATE


Titillate means “stimulate pleasantly” or “tickle.” Titivate means “adorn” or
“spruce up.” The similarity in spelling is coincidental. Titillate comes from the Latin
word titillare, which means “tickle. Titivate is a newer word which comes from the
earlier word tidivate, which is likely derived from the words tidy and elevate.

TORTUOUS AND TORTUROUS


Tortuous means full of twists and turns. Torturous means “causing or involving
torture or suffering.” These words are sometimes confused because of their similar
spelling, but there is no further similarity.

TURBID AND TURGID


Turbid means “unclear, obscure, confused, disordered” and, when referring to
liquids, “muddy, thick, unclear.” Turgid means “enlarged, swollen” and, when
referring to language, “pompous, overblown grandiloquent.” Because the two words
are relatively uncommon, one is often confused for the other. Turbid comes from the
Latin word turbidus, meaning “disordered,” from turba, which means turmoil,
possibly from the Greek word turbe. Turgid comes from the Latin word turgidus,
from turgere, which means, “to be swollen.”

VENAL AND VENIAL


Venal means “open to bribery” and “corruptible” and, when referring to conduct,
“influenced by bribery.” Venial means “pardonable,” when referring to a fault or
sin, and “trifling, not serious” when referring to misconduct.

127
7
Synonyms

Following are some groups of near synonyms. The words in a group have
almost similar meaning, yet a sophisticated user of English would not use one for
another owing to slight variance in their meaning. It is suggested to you that you
should go through the list to imbibe the nuances in the use of synonyms.
1.
caricature refers to an imitation or representation of a person or
thing, in drawing, writing, or performance, that ludicrously
exaggerates its distinguishing features;
Burlesque implies the handling of a serious subject lightly or
flippantly, or of a trifling subject with mock seriousness;
a parody ridicules a written work or writer by imitating the style
closely, esp. so as to point out its peculiarities or
affectations, and by distorting the content nonsensically
or changing it to something absurdly incongruous;
travesty in contrast, implies that the subject matter is retained, but
that the style and language are changed so as to give a
grotesquely absurd effect;
satire refers to a literary composition in which follies, vices,
stupidities, and abuses in life are held up to ridicule and
contempt;
lampoon refers to a piece of strongly satirical writing that uses
broad humor in attacking and ridiculing the faults and
weaknesses of an individual
2.
an epicure is a person who has a highly refined taste for fine foods
and drinks and takes great pleasure in indulging it;
SYNONYMS

a gourmet is a connoisseur in eating and drinking who appreciates


subtle differences in flavor or quality;
gourmand occasionally equivalent to gourmet, is more often applied
to a person who has a hearty liking for good food or one
who is inclined to eat to excess;
a gastronome is an expert in all phases of the art or science of good
eating;
a glutton is a greedy, voracious eater and drinker
3.
severe applies to a person or thing that is strict and
uncompromising and connotes a total absence of softness,
laxity, frivolity, etc. [a severe critic, hairdo, etc.];
stern implies an unyielding firmness, esp. as manifested in a
grim or forbidding aspect or manner [a stern guardian];
austere suggests harsh restraint, self-denial, stark simplicity [the
austere diet of wartime], or an absence of warmth, passion,
ornamentation, etc. [an austere bedroom];
ascetic implies extreme self-denial and self-discipline or even,
sometimes, the deliberate self-infliction of pain and
discomfort, as by religious fanatics [an ascetic hermit]
—ANT. mild, lax, indulgent
4.
range refers to the full extent over which something is
perceivable, effective, etc. [the range of his knowledge];
reach refers to the furthest limit of effectiveness, influence, etc.
[beyond the reach of my understanding];
scope implies considerable room and freedom of range, but
within prescribed limits [does it fall within the scope of
this dictionary?];
compass also suggests completeness within limits regarded as a
circumference [he did all within the compass of his power];

129
ENGLISH USAGE

gamut in this connection, refers to the full range of shades, tones,


etc. between the limits of something [the full gamut of
emotions]
5.
circumference refers to the line bounding a circle or any approximately
circular or elliptical area;
perimeter extends the meaning to a line bounding any area, as a
triangle, square, or polygon;
periphery in its literal sense identical with perimeter, is more
frequently used of the edge of a physical object or in an
extended metaphoric sense [the periphery of
understanding];
circuit now usually refers to a traveling around a periphery [the
moon‘s circuit of the earth];
compass refers literally to an area within specific limits but is often
used figuratively [the compass of the city, the compass of
freedom]
6.
despise implies a strong emotional response towards which one
looks down upon with contempt or aversion [to despise
a hypocrite];
to scorn is to feel indignation toward or deep contempt for [to
scorn the offer of a bribe];
disdain implies a haughty or arrogant contempt for what one
considers beneath one‘s dignity [to disdain flattery];
contemn chiefly a literary word, implies a vehement disapproval of
a person or thing as base, vile, or despicable
7.
hate implies a feeling of great dislike or aversion, and, with
persons as the object, connotes the bearing of malice;
detest implies vehement dislike or antipathy;

130
SYNONYMS

despise suggests a looking down with great contempt upon the


person or thing one hates;
abhor implies a feeling of great repugnance or disgust;
loathe implies utter abhorrence —ANT. love, like
8.
love implies intense fondness or deep devotion and may apply
to various relationships or objects [sexual love, brotherly
love, love of one‘s work, etc.];
affection suggests warm, tender feelings, usually not as powerful
or deep as those implied by love [he has no affection for
children];
attachment implies connection by ties of affection, attraction,
devotion, etc. and may be felt for inanimate things as well
as for people [an attachment to an old hat];
infatuation implies a foolish or unreasoning passion or affection,
often a transient one [an elderly man‘s infatuation for a
young girl]
9.
murmur implies a continuous flow of words or sounds in a low,
indistinct voice and may apply to utterances of satisfaction
or dissatisfaction [to murmur a prayer];
mutter usually suggests angry or discontented words or sounds
of this kind [to mutter curses]; to
mumble is to utter almost inaudible or inarticulate sounds in low
tones, with the mouth nearly closed [an old woman
mumbling to herself]
10.
sad is the simple, general term, ranging in implication from a
mild, momentary unhappiness to a feeling of intense grief;
sorrowful implies a sadness caused by some specific loss,
disappointment, etc. [her death left him sorrowful];

131
ENGLISH USAGE

melancholy suggests a more or less chronic mournfulness or


gloominess, or, often, merely a wistful pensiveness
[melancholy thoughts about the future];
dejected implies discouragement or a sinking of spirits, as because
of frustration;
depressed suggests a mood of brooding despondency, as because
of fatigue or a sense of futility [the novel left him feeling
depressed];
doleful implies a mournful, often lugubrious, sadness [the doleful
look on a lost child‘s face] —ANT. happy, cheerful
11.
happy generally suggests a feeling of great pleasure,
contentment, etc. [a happy marriage];
glad implies more strongly an exultant feeling of joy [your letter
made her so glad], but both glad and happy are commonly
used in merely polite for mulas expressing gratification
[I‘m glad, or happy, to have met you];
cheerful implies a steady display of bright spirits, optimism, etc.
[he‘s always cheerful in the morning];
joyful and joyous both imply great elation and rejoicing, the former generally
because of a particular event, and the latter as a matter of
usual temperament [the joyful throngs, a joyous family]
—ANT. sad
12.
accidental describes that which occurs by chance [an accidental
encounter] or outside the normal course of events [an
accidental attribute];
fortuitous which frequently suggests a complete absence of cause,
now usually refers to chance events of a fortunate nature;
casual describes the unpremeditated, random, informal, or
irregular quality of something [a casual visit, remark, dress,
etc.];

132
SYNONYMS

incidental emphasizes the nonessential or secondary nature of


something [an incidental consideration];
adventitious refers to that which is added extrinsically and connotes a
lack of essential connection
13.
destroy implies a tearing down or bringing to an end by wrecking,
ruining, killing, eradicating, etc. and is the term of broadest
application here [to destroy a city, one‘s influence, etc.];
demolish implies such destructive force as to completely smash to
pieces [the bombs demolished the factories];
raze means to level to the ground, either destructively or by
systematic wrecking with a salvaging of useful parts;
to annihilate is to destroy so completely as to blot out of existence
[rights that cannot be annihilated]
14.
poverty the broadest of these terms, implies a lack of the resources
for reasonably comfortable living;
destitution imply such great poverty that the means for mere
and want subsistence, such as food and shelter, are lacking;
indigence a somewhat euphemistic term, implies a lack of luxuries
which one formerly enjoyed;
penury suggests such severe poverty as to cause misery, or a
loss of self-respect—ANT. wealth, affluence
15.
crowd is applied to an assembly of persons or things in close
proximity or densely packed together and may suggest
lack of order, loss of personal identity, etc. [crowds lined
the street];
throng specifically suggests a moving crowd of people pushing
one another [throngs of celebrators at Times Square];
multitude stresses greatness of number in referring to persons or
things assembled or considered together [a multitude
arrayed against him];

133
ENGLISH USAGE

swarm suggests a large, continuously moving group [a swarm


of sightseers];
mob properly applied to a disorderly or lawless crowd, is an
abusive term when used to describe the masses or any
specific group of people;
host specifically suggests a large organized body marshaled
together but may be used generally of any sizable group
considered collectively [he has a host of friends];
horde specifically refers to any large predatory band [a horde of
office seekers]
16.
food is the general term for all matter that is taken into the
body for nourishment;
fare refers to the range of foods eaten by a particular organism
or available at a particular time and place [the fare of
horses, a bill of fare];
victuals is a dialectal or colloquial word for human fare or diet;
provisions in this connection, refers to a stock of food assembled in
advance [provisions for the hike];
ration refers to a fixed allowance or allotment of food [the weekly
ration] and in the plural (rations) to food in general [how
are the rations in this outfit?]
17.
curse is the general word for calling down evil or injury on
someone or something;
damn carries the same general meaning but, in strict usage,
implies the use of the word “damn” in the curse [he
damned his enemies = he said, “Damn my enemies!”];
execrate suggests cursing prompted by great anger or abhorrence;
imprecate suggests the calling down of calamity on someone, esp.
from a desire for revenge;

134
SYNONYMS

anathematize strictly refers to the formal utterance of solemn


condemnation by ecclesiastical authority, but in general
use it is equival1 —ANT. bless
18.
scold is the common term meaning to find fault with or rebuke
in angry, irritated, often nagging language [a mother
scolds a naughty child];
upbraid implies bitter reproach or censure and usually connotes
justification for this [she upbraided me for my
carelessness];
berate suggests continuous, heated, even violent reproach, often
connoting excessive abuse [the old shrew continued
berating them];
revile implies the use of highly abusive and contemptuous
language and often connotes deliberate defamation or
slander [he reviled his opponent unmercifully];
vituperate suggests even greater violence in the attack [vituperating
each other with foul epithets]
19.
criticize in this comparison, is the general term for finding fault
with or disapproving of a person or thing;
reprehend suggests sharp or severe disapproval, generally of faults,
errors, etc. rather than of persons;
blame stresses the fixing of responsibility for an error, fault, etc.;
censure implies the expression of severe criticism or disapproval
by a person in authority or in a position to pass judgment;
condemn and denounce both imply an emphatic
pronouncement of blame or guilt,
condemn suggesting the rendering of a judicial decision, and
denounce, public accusation against persons or their
acts—ANT. Praise

135
ENGLISH USAGE

20.
praise is the simple, basic word implying an expression of
approval, esteem, or commendation [to praise one‘s
performance];
laud implies great, sometimes extravagant praise [the critics
lauded the actor to the skies];
acclaim suggests an outward show of strong approval, as by
loud applause, cheering, etc. [he was acclaimed the
victor];
extol implies exalting or lofty praise [the scientist was extolled
for his work];
eulogize suggests formal praise in speech or writing, as on a special
occasion [the minister eulogized the exemplary life of the
deceased]
21.
deception is applied to anything that deceives, whether by design
or illusion;
fraud suggests deliberate deception in dishonestly depriving a
person of property, rights, etc.; subterfuge suggests an
artifice or stratagem used to hide one’s true objective, to
evade something, or to gain some end;
trickery implies the use of tricks or ruses in deceiving others;
chicanery implies the use of clever but tricky talk or action, esp. in
legal actions
22.
female is the basic term applied to members of the sex that is
biologically distinguished from the male sex and is used
of animals or plants as well as of human beings;
feminine is now the preferred term for references, other than those
basically biological, to qualities thought to be
characteristic of or suitable to women, as delicacy,
gentleness, etc.;

136
SYNONYMS

womanly suggests the noble qualities one associates with a woman,


esp. one who has maturity of character;
womanish in contrast, suggests the weaknesses and faults that are
regarded as characteristic of women;
effeminate used chiefly in reference to a man, implies delicacy,
softness, or lack of virility;
ladylike refers to manners, conduct, etc. such as are expected from
a refined or well-bred woman
23.
male is the basic term applied to members of the sex that is
biologically distinguished from the female sex and is used
of animals and plants as well as of human beings;
masculine is applied to qualities, such as strength and vigor,
traditionally ascribed to men, or to things appropriate to
men;
manly suggests the noble qualities, such as courage and
independence, that a culture ideally associates with a
man who has maturity of character;
mannish used chiefly of women, is most often used derogatorily
and implies the possession or adoption of the traits and
manners thought to be more appro priate to a man;
virile stresses qualities such as robustness, vigor, and, spefic,
sexual potency, that belong to a physically mature man
24.
superficial implies concern with the obvious or surface aspects of a
thing [superficial characteristics] and, in a derogatory
sense, lack of thoroughness, profoundness, significance,
etc. [superficial judgments];
shallow in this connection always derogatory, implies a lack of
depth of character, intellect, meaning, etc. [shallow
writing];
cursory which may or may not be derogatory, suggests a hasty
consideration of something without pausing to note
details [a cursory inspection]
137
ENGLISH USAGE

ANSWERS
EXERCISE - 1
1. (1) 2. (2) 3. (2) 4. (4) 5. (4)
6. (2) 7. (3) 8. (2) 9. (1) 10. (3)

EXERCISE - 2
1. (2) 2. (4) 3. (3) 4. (2) 5. (3)
6. (3) 7. (4) 8. (2) 9. (2) 10. (2)
11. (1) 12. (1) 13. (3) 14. (4) 15. (1)
16. (4) 17. (3) 18. (4)

EXERCISE - 3
1. (2) 2. (2) 3. (3) 4. (1) 5. (4)
6. (3) 7. (1) 8. (1) 9. (2) 10. (4)
11. (4) 12. (1) 13. (2) 14. (4) 15. (4)

EXERCISE - 4
1. (3) 2. (4) 3. (2) 4. (4) 5. (4)
6. (2) 7. (2) 8. (3) 9. (4) 10. (2)
11. (3) 12. (2) 13. (1) 14. (3) 15. (3)
16. (1) 17. (4) 18. (1) 19. (2) 20. (4)

EXERCISE - 5
1. (4) 2. (1) 3. (4) 4. (4) 5. (2)

EXERCISE - 6
1. (4) 2. (1) 3. (3) 4. (1) 5. (4)

138
ANSWERS

EXERCISE - 7
1. (3) 2. (2) 3. (4) 4. (3) 5. (2)
6. (4) 7. (1) 8. (3) 9. (3) 10. (2)
11. (3) 12. (2) 13. (4) 14. (2) 15. (1)
16. (3) 17. (1) 18. (2) 19. (4) 20. (3)
21. (2) 22. (1) 23. (2) 24. (3) 25. (1)

EXERCISE - 8
1. (3) 2. (1) 3. (2) 4. (1) 5. (3)

EXERCISE - 9
1. (3) 2. (4) 3. (3) 4. (1) 5. (2)

EXERCISE - 10
1. (3) 2. (2) 3. (2) 4. (1) 5. (3)
6. (3) 7. (3) 8. (4) 9. (3) 10. (2)
11. (4) 12. (3) 13. (2) 14. (3) 15. (3)
16. (3) 17. (3) 18. (1) 19. (4) 20. (3)

EXERCISE - 11
1. (2) 2. (4) 3. (3) 4. (3) 5. (3)
6. (2) 7. (2) 8. (3) 9. (4) 10. (2)
11. (3) 12. (3) 13. (3) 14. (1) 15. (2)
16. (4) 17. (3) 18. (4) 19. (4) 20. (4)
21. (4) 22. (4) 23. (2) 24. (1) 25. (4)

139
ENGLISH USAGE

EXERCISE - 12
1. (2) 2. (1) 3. (1) 4. (2) 5. (1)
6. (4) 7. (1) 8. (2) 9. (3) 10. (4)
11. (4) 12. (4) 13. (2) 14. (4) 15. (1)
16. (4) 17. (3) 18. (2) 19. (4) 20. (3)
21. (2) 22. (3) 23. (2) 24. (2) 25. (4)

EXERCISE - 13
1. (3) 2. (1) 3. (2) 4. (2) 5. (3)

EXERCISE - 14
1. (1) 2. (3) 3. (4) 4. (2) 5. (4)
6. (4) 7. (3) 8. (4) 9. (2) 10. (4)

EXERCISE - 15
1. (2) 2. (3) 3. (2) 4. (1) 5. (4)
6. (1) 7. (4) 8. (1) 9. (2) 10. (3)

EXERCISE - 16
1. (3) 2. (1) 3. (2) 4. (3) 5. (4)
6. (4) 7. (3) 8. (2) 9. (2) 10. (3)
11. (2) 12. (4) 13. (3) 14. (1) 15. (4)
16. (4) 17. (4) 18. (1) 19. (1) 20. (1)

140
EXPLANATIONS

EXPLANATIONS
EXERCISE – 1
1. Answer choices 3] and 4] weaken the analyst’s argument. Choice 2] is not
conclusive because it indicates the patent is on a new product and we do not
know if the patent is for a product consumers will demand. Answer choice 1] is
the best one available. If the revenues increase and the expenses decrease,
then the company can significantly increase its profits and be more likely to
enjoy a high stock price appreciation rate. So, the answer is 1]
2. Answer choice 4] actually strengthens the argument – which is the opposite of
what the question asks you to do. Answer choice 3] is completely irrelevant to
the argument. Acme could still be the best product, even if most people don’t
intend to use it. Choice 1] certainly does not bode well for the quality of Acme’s
aspirin, but it is nonetheless possible that competing brands are even
more.Choice 2] is correct. It weakens the passage’s argument by undercutting
its implied assumption that a sample of 10 people must give accurate results.
So, the answer is 2]
3. Answer choice 2] can be easily, directly, and correctly inferred from the statement
that the dogs bark and howl every time their owner lets them outside. The rest
of this passage is “decoy” material. The writer has used two tricks to make you
deviate from the correct answer. The first trick is to have an unqualified authority
comment on a controversy. In this case, we have no reason to believe a CPA
would be an expert on canine behavior. The second trick is to present extreme
or emotionally loaded statements. Answer choice (1) is too extreme to be a correct
answer choice. So, the answer is 2]
4. Choice 4] is correct because it neatly summarizes the main theme of the passage.
We can instantly eliminate answer choice (1) because the passage never states
how the increased penalty affects citizens. Choice 2] is too extreme, and overly
negative. Besides, the overall theme of the passage is positive; the negative
tone of choice 2] is out of keeping with it. Answer choice 3] is incorrect because
the author stated only that adherence to the law would prevent the price of
gasoline from rising further. He never said that this action would lower gas
prices. So, the answer is 4]

141
ENGLISH USAGE

5. Option 1] goes against the opinion of Yasukuni. He holds that the aspirants
should primarily focus on the seat in Security counsel. Veto power should not
be made the central issue. Though it may be important, in case of opposition,
the aspirants should adopt a flexible attitude about it. So, the answer is 4]
6. The word ‘unfortunate’ in the first sentence in the paragraph indicates clearly that
the shift in India’s stance to Nepal is not a happy one. Without sharing a very
cordial relation, India is bound to engage with Nepal due to different reasons
mentioned in the paragraph. Option 2] suggests clearly this attitude. If India
does not help Nepal, then China or Pakistan can and this will increase vulnerability
on Indian border. So, the answer is 2]
7. The question asks you the possible opinion of the critics which could be the
one in option 3]. Mark the word ‘yet’ in the second sentence of the paragraph.
The tone suggests that the author is holding the view which is opposite to the
one of the critics. The author says that the present Pope is ‘not going to be the
carbon copy of his predecessor’. It means that the critics think that he is. So,
the answer is 3]
8. Economic growth could be a one of the pretexts but it is not only a ploy to
divert public attention. Let us rule out 1] then. The author is not suggesting
what is given in 3]. Though one can infer option 4] as well but the paragraph is
not substantiating this assumption. Whereas, there is no doubt about the
author’s opinion that the concentration of wealth leads to more unjust
distribution. So, the answer is 2]
9. The tone of the author is evidently not in favour of the implementation of
POTA. As mentioned in option 2], the author is apprehensive about the failure
the of witness to testify. But it is not the reason of authors distaste, nor is he
desirous of improvement in this provision as mentioned in option 3]. In the last
line the author criticizes the government for not reviewing the ‘draconian’
provisions. Obviously, he shares the view of the critics. So, the answer is 1]
10. The court does not allow the press to distort the story, nor is it remotely
suggested that an organization can be harmed in the benefit of society. Thus,
options 1] and 2] can be eliminated. The statement in option 4] has no bearing
with the passage. Option 3], however, is in line with the court order. So, the
option is 3].

142
EXPLANATIONS

EXERCISE – 3
1. C is apparently the opening sentence here. Then we have two possibilities, 2]
and 4]. B tells the cause and D is the consequence of the problem. Thus D will
follow B. So, the answer is 2]
2. The ‘globalization’ mentioned in B has been rejected that Friendman cherishes
in his book mentioned in C. Hence, B has to follow C. So, the answer is 2]
3. D opens the theme with a general proposition. The apparent link of D is B. So,
the answer is 3]
4. A leads to D. We have two possibilities then, 1] and 2]. But B is not leading to
C, as given in 2]. So, the answer is 1]
5. The argument here goes from general to specific. First, you have ‘four leaders’
in D who discuss the problem of Germany. C mentions two leaders expressing
their views. B states about Antony’s speech. Understandably, the answer is 4]
6. C is leading to D and A is leading to B. The statement clearly opens with the
general sentence C. So, the answer is 3]
7. A introduces the theme which is built on further by D. So, the answer is 1]
8. The opening sentence is clear which is A. We have two possibilities 1] and 2].
Again we see that C leads naturally to D. 2] is ruled out then. So, the answer is
1]
9. The expositions of theme ‘feeling of insecurity’ we find in D. ABC tell us how to
cope with the problem. So, the answer is 2]
10. The question is too simple to need any explanation. The sequence of time itself
can get you the solution. So, the answer is 3]
11. Both A and C seems to be exposing the theme, as they tell more or less the same
thing. D is leading to A. So the answer is 4]
12. B and D state the positive consequences, whereas A and C negative. Both the
pairs have to be in consecutive order. Only 1] follows this order. So the answer
is 1]
13. In chronological order, the narrative states a historical mishap. Unmistakably
the opening sentence is D. So, the answer is 2]
14. ‘They’ in A represents ‘Officials’ in B. Thus, A has to follow B. Again India in C
could also become a ‘global player’ like the U.S., Russia and France in A. So, the
answer is 3]
15. A leads to B. We have options 1] and 4] with AB sequence. But A is not the
opening sentence. So, the answer is 4]

143
ENGLISH USAGE

EXERCISE – 5
1. The political end by using religion can be achieved by bringing the religious
community together mobilizing it. The other effects listed in the options are
irrelevant in this context. So, the answer is 4]
2. Walking on the ramps is a career for models. The word ‘alternative’ makes it
sure that models are going for more career options besides modeling. So, the
answer is 1]
3. The show of religiosity has helped separatism. Options 1] and 2] are ruled out
then, as they are negative in nature. Again, ‘celebrated’ will be too strong a
word. So, the answer is 4]
4. Employment is an economic issue and it is a direct consequence of economic
development. So, the answer is 4]
5. A greedy person is characterized by his love of material possessions. So, the
answer is 2]

EXERCISE – 7
1. 1. After reading the sentence you come to know that it is ‘necessary to
choose between competing alternatives’. Evidently it points to the careful
use of the resources because they are limited. So, the answer is 3]
2. There are two kinds of economic systems; market oriented (capitalistic) and
planned (socialistic). The sentence mentions both types. So the answer is 2]
3. Market oriented economy largely depends on the demands by the
consumer, which are the manifestations of their desires. So, the answer is
4]
4. In the latter part of the sentence you have to find within quotes the
questions a agent (consumer) might deliberate. There is no doubet here
that it tells of choices that relate to one’s income and so on. So, the
answer is 3]
5. After such deliberations an agent learns to ‘select’ well among his choices.
So, the answer is 2]
2. 6. The first sentence of the paragraph mentions that ‘archetype’ is the original
model. So, the answer is 2]

144
EXPLANATIONS

7. Shakespeare, the great master of literature, has portrayed many characters


who, along with having unique individual characteristics, also express
emotions which are universal and archetypel. So, the answer is 1]
8. Shakeshpeare is related to literature. So, the answer is 4]
9. As mentioned in explanation 2. Shakeshpearean characters appear original
and unique individuals inspite of being archetypal.
10. Pastiche is concerned to the art of mimicry. So, the answer is 2]
3. 11. The next sentence mentions the distinction between natural and artifical
as a ‘problem’; that is, it is difficult to distinguish between them. So, the
answer is 3]
12. Since it is a problem, it has to be solved. So, the answer is 2]
13. The context of the paragraph clearly suggests that the distinction is not
clear. So the answer is 4]
14. Any artifice is the result of human action. So the answer is 2]
15. The sentence is about philosophy. So, the answer is 1]
4. 16. The words ‘syllogisms’ and ‘logic’ suggest that problem are posed in a
‘rational’ manner. So the answer is 3]
17. Critical reading and reasoning have close meaning. So, the answer is 1]
18. Among all options, personal ‘contemplation’ could be the best way of
finding an answer. Other options do not fit here. So, the answer is 2]
19. After reading further you will find that the role of language in discources
is discussed. So, the answer is 4]
20. Philosophy involves acts of deliberations and contemplations. So, the
answer is 3]
5. 21. It is clear that the expansion of the Westerners must have ‘expelled’ the
Native Americans.
22. In the previous sentence we know that the Native Americans have been
pushed westward. So the answer is 1]
23. The conflict is between the colonizer and the colonized, the US and the
local Native American tribes. So the answer is 2]

145
ENGLISH USAGE

24. Mark the connector ‘but’ in the sentence. It suggests that the treaties
could not fulfill their purpose. They were abrogated. So, the answer is 3]
25. The sentence is about war. It involves massacre. So the answer is 1]

EXERCISE – 8
1. After careful reading of the paragraph, you will find that it is discussing the
consequences in the present of the economic decisions Japan took after the
WW2. The sentence after the blank space contains phrase ‘less ideological
and planned approach’, indicating that the decisions were taken haphazardly.
So, the answer is 3]
2. It is clear from the first two sentences that the writer holds lacuna in the
liberalization programme responsible for the Gurgaon incident. We find that the
option 1] states that management of companies find loopholes in the company
laws. Means it is building on the previous theme. So, the answer is 1]
3. The sentences immediately before and after the blank space are elucidating
how the people from one sect would let people another sect live in places which
they consider sacred. Clearly, the missing sentence should also be of the same
nature. So, the answer is 2]
4. 2] does not appear relevant and as the paragraph mentions ‘despite possessing
purchasing power’, so we can rule out 3] as well. Option 4] too cannot be the
answer because it should have come as the third sentence in the paragraph to
provide proper sequence. So, the answer is 1]
5. The last sentence of the paragraph shows attitude of the Westerners towards
Indian music. Option 3] is visibly providing the link. So, the answer is 3].

EXERCISE – 14
1. Options 2, 3 and 4 are all synonymous for ‘lie’ or dishonesty, while veracity
means truthfullness. So the answer is 1].
2. Rogue, Vagabond and mendicant are those who donot have a permanent
dwelling, while a ‘hooligan’ is an aggressive violent youth. So the answer is 3].
3. Options 1, 2 and 3 are all synonymous for ‘dull’, whereas Arduous means hard-
working. So the answer is 4].

146
EXPLANATIONS

4. Options 1, 3 and 4 mean touching something from outside (peripherial) while


basic means to go inside. So the answer is 2].
5. Options 1, 2, 3 are related with altering of facts, whereas expedition is something
totally different. So the answer is 4].
6. Goad means ‘to urge’, while all the others options are related with ill-treatment.
So the answer is 4].
7. Plato, Aristotle and Socrates were philosphers while Einstein was a scientist.
So the answer is 3].
8. Atheist is a non believer of God, while all the others are Religious. So the
answer is 4].
9. Option 1, 3 and 4 are sociable, while Reticent is a reseved person. So the answer
is 2].
10. Option 1, 2 and 3 are nelated with horse in some way or the other, while grant is
not, so the answer is 4].

147

You might also like